Algebra Basica

Page 1


David Gonzáles López

ÁLGEBRA BÁSICA Teoría y práctica


ÁLGEBRA BÁSICA Teoría y práctica © David Gonzáles López Primera Edición Lambayeque, diciembre 2009 Impreso en Impresiones Montenegro Calle Manco Cápac 485 - Chiclayo 500 ejemplares

Hecho el Depósito Legal en la Biblioteca Nacional del Perú Nº 2009 - 16362

Prohibida la reproducción total o parcial de esta obra por cualquier medio, sin la autorización escrita del autor.


Presentación El Álgebra es el lenguaje de las matemáticas y una de sus ramas que estudia a la cantidad del modo más general posible. Las matemáticas son, esencialmente, la expresión o reducción de ideas complejas y sofisticadas mediante símbolos, y operaciones sobre símbolos. Una vez que tenemos los símbolos y las operaciones aparece el álgebra. El Álgebra tiene por objeto simplificar, generalizar y resolver las cuestiones relativas a la cantidad, determinando las operaciones que hay que efectuar para llegar a cierto resultado, transformando las expresiones algebraicas en otras equivalentes y adquiriendo las bases para mas tarde poder hacer planteamientos matemáticos que representen la realidad. El dominio del Álgebra elemental tiene una enorme importancia para el dominio de la matemática porque en él se conjugan capacidades, habilidades y destrezas; ya que en cada uno de sus temas el alumno deberá poner en juego un alto grado de práctica y abstracción. El propósito de este material es hacer llegar a los postulantes a universidades y centros de estudios superiores el desarrollo teórico, práctico y formativo de algunos temas del álgebra, los cuales son muy necesarios y relevantes para el aprendizaje del álgebra y la matemática en general. Sirve también como material de consulta para los estudiantes de cursos avanzados de matemáticas. Cada tema desarrollado tiene parte teórica, ejercicios resueltos y propuestos para una cabal retroalimentación. Estos temas se han desarrollado minuciosamente, para una fácil comprensión por el lector; en muchos casos, se ha optado por dos o más formas de solución, dándole al estudiante un mayor panorama en cuanto los criterios a tomar frente a un problema determinado. Espero que este material logre convertirse en un importante auxiliar pedagógico para todos los estudiantes egresados de secundaria; y a través de él, logre aportar en su preparación preuniversitaria y en su posterior desarrollo profesional.

El Autor

ÍNDICE


Presentación 1.1. CONCEPTOS PREVIOS ……………………………………………………………………… 01 Conjuntos numéricos / El conjunto de los números naturales / El conjunto de los números enteros / El conjunto de los números racionales / El conjunto de los números irracionales / El conjunto de los números reales / El conjunto de los números complejos. 1.2. EXPRESIONES ALGEBRAICAS …………………………………………………................. 04 Álgebra / Expresiones algebraicas / término algebraico / términos semejantes / clasificación de las expresiones algebraicas / Grado de una expresión algebraica / Polinomios especiales / valor numérico de una expresión algebraica. 1.3. OPERACIONES CON POLINOMIOS ENTEROS …………………………………………... 11 Adición y sustracción de polinomios / Multiplicación de polinomios / Productos notables / División de polinomios / Teorema del resto / Teorema del factor / Cocientes notables. 1.4. FACTORIZACIÓN …………………………………………………………………. ... ………... 27 Polinomio primo sobre un conjunto numérico / Métodos para factorizar una expresión algebraica. 1.5. FRACCIONES ALGEBRAICAS RACIONALES ……………………………………………. 42 Fracción algebraica / Clases de fracciones / signos de una fracción / simplificación de fracciones/ Operaciones con fracciones / simplificación de fracciones complejas / Fracciones parciales. 1.6. TEORÍA DE EXPONENTES ……………………………………………………….. ………… 58 Potenciación / Radicación / Leyes de exponentes. 1.7. RACIONALIZACIÓN ……………………………………………………………….. .. ………… 66 Factor racionalizante / casos que se presentan para racionalizar.


DAVID GONZÁLES LÓPEZ

ÁLGEBRA BÁSICA - TEORÍA Y PRÁCTICA 1.1. CONCEPTOS PREVIOS Conjuntos numéricos Los conjuntos numéricos que se estudian en las matemáticas son: Los números naturales, números enteros, números racionales, números irracionales, números reales y números complejos. El conjunto de los números naturales ( N ) Es el conjunto denotado por N cuyos elementos son empleados para realizar la operación de contar.

N = { 0, 1, 2, 3, 4, 5, 6, 7, 8, 9, 10, 11, . . . , n , . . . } N = { x / x es un número natural }= { x / x ∈ N} El conjunto de los números enteros ( Z ) Contar con números más y más grandes no era problema, contar en forma descendente era asunto distinto : 5, 4, 3, 2, 1, 0 ¿Pero qué venía después de cero?. Sin embargo, si se habla de deudas, temperaturas muy frías y aún de las cuentas regresivas en los lanzamientos a la luna, debemos tener una respuesta. Enfrentándose a este problema, los matemáticos inventaron un cúmulo de números: ---1, -2, -3, -4, -5, ... llamados enteros negativos, que junto con los números naturales forman el conjunto de los números enteros, denotado por Ζ .

Ζ = Ζ =

{ . . . , - n , . . ., - 3, - 2, - 1, 0 , 1, 2, 3, 4, . . ., n , . . . } { x / x es un número entero } = { x / x ∈ Ζ }

Del conjunto Ζ podemos obtener los siguientes subconjuntos :

{ . . . , - 4, - 3, - 2, -1 } Ζ = { 1, 2, 3, 4, . . . } Ζ 0− = { . . . , - 4, - 3, - 2, -1, 0 } Ζ 0+ = { 0, 1, 2, 3, 4, . . . } Z− = +

El conjunto de los números racionales ( Q ) Enfrentados a la necesidad de dividir, los matemáticos decidieron que el resultado de dividir un entero entre otro entero distinto de cero se podía ver como un número. Esto significa que :

3 7 - 2 14 8 , , , , , etc. son números con todos los derechos y privilegios de los enteros e 4 8 3 - 15 2 inclusive un poco más. Siempre es posible dividir excepto entre cero. De manera natural esos números se llaman números racionales (una razón de números). El conjunto de los números racionales se denota por Q .

1


DAVID GONZÁLES LÓPEZ

m 1 1 1 m   Q =  ... , , ... , -1, .. , - , ... , - , ... , 0 , ... , , ... ,1, ... , 5 , ... , , ...  n 2 3 2 n   m   Q =  x / x = , m , n∈Ζ , n ≠ 0  = { x / x∈ Q } n   ¿Cuándo un número decimal es racional? a) Los números decimales finitos son racionales

0,23 =

23 ∈Q 100

b) Los números decimales periódicos puros y periódicos mixtos son números racionales.

3 1 = ∈Q 9 3 21 7 0,212121... = = ∈Q 99 33

0,3333... =

241 − 2 239 = ∈Q 990 990 232 − 2 122 1,2323232... = 1 + = ∈Q 990 99 0,2414141... =

El conjunto de los números Irracionales ( I ) Está formado por todos los números decimales que no se pueden expresar de la forma

m con n

m, n ∈ Z y n ≠ 0 . El conjunto de los números irracionales se denota por I .

{

}

I = ...,-π ,...,- 5 ,...,−3 2 ,..., 3 ,..., 5 ,..., π ,...,4e ,... I = { x / x tiene representación decimal infinita no periódica } = { x/ x ∈ I } Hay dos números irracionales muy importantes en las matemáticas, dichos números son: a) El número pi (π ) , cuya aproximación decimal es: π ≈ 3,1416 Una primera referencia del valor de π aparece en la Biblia. En el primer libro de los Reyes, capítulo 7, versículo 23. Aquí el valor de π es 3, inexacto por supuesto. El número π se obtiene de la relación que existe entre la longitud de una circunferencia y su diámetro, es decir:

π=

C C : Longitud de la circunferencia = 2R R : Radio de la circunferencia

b) El número e (épsilon ), cuya aproximación decimal es: e = 2,7182 El conjunto de los números Reales ( R ) El conjunto de los números reales denotado por R , es la reunión de los números naturales, enteros, racionales e irracionales Así : R = N ∪ Z ∪ Q ∪ I , además Q ∩ I = φ . También R = Q ∪ I . Intuitivamente los números reales se representa por una recta y la llamamos RECTA REAL

2


DAVID GONZÁLES LÓPEZ

3   I =  ...,-π ,...,- 5 ,...,−2 ,...,−1 ,...,0 ,...,1,..., ,..., 2 ,..., 5 ,..., 28 ,... 2   I = { x / x es un número racional o irracional } = { x/ x ∈ R } El conjunto de los números Complejos ( C ) Sean las ecuaciones x2 +1 = 0 y x2 + 4 = 0 desarrollando se tiene

x 2 = −1 x = ± −1 x 2 = ±1

x2 + 4 = 0 x = ± −4 x = ±2i

Ambas ecuaciones no tienen solución en el conjunto de los números reales. Frente a esta situación aparece el número i = − 1 que satisface i 2 = − 1 .Descartes fue el primero en llamarlo número imaginario. Es así como aparecen los números complejos:

2 1 i, + 4i, 7 - 2i, 5 + 0i, 3 - 2 i , etc. 3 2 Los números complejos se denotan por C . C = x / x = a + bi ; a, b ∈ R, i = - 1 = { x / x ∈ C } 2i, - 4i, 3 + 2i,

{

}

Los números complejos se representan de la siguiente forma a + bi : forma binómica (a , b) : forma cartesiana , donde a : parte real b : parte imaginaria Conclusión - Todo número es complejo - N⊂Z⊂Q⊂R ⊂C - I es disjunto de N, Z y Q Gráfica

3


DAVID GONZÁLES LÓPEZ

1.2. EXPRESIONES ALGEBRAICAS Álgebra Parte de la matemática elemental que estudia a las cantidades en su forma mas general, haciendo uso para ello de letras y números. Teniendo como objetivo transformar, generalizar, simplificar y resolver cuestiones relativas a la cantidad. Expresiones algebraicas Llamamos expresión algebraica a toda combinación de números y letras (variables) unidas entre si por los signos de diferentes operaciones aritméticas: adición, sustracción, multiplicación, división, potenciación y radicación. Ejemplos: xy , 2x 3 , 2x 2 y − 5 y ,

6x 3 − x 2 − 5 4x 3 yz 2 , x-y 3x 2 − 6 y

No son expresiones algebraicas: 8 x , cos x , log 4 x Observación Los tres últimos ejemplos son expresiones trascendentes o no algebraicas, denominadas: exponencial, trigonométrica y logarítmica respectivamente. Término algebraico Es aquella expresión algebraica donde no se encuentran presentes las operaciones de adición y sustracción. Ejemplo: coeficiente

+ 2x 3 signo

exponente parte literal

Otros ejemplos : 7 x −3 , − 3x −2 y , - 4xy1/2 , x y 3 z , Términos semejantes Son aquellos términos que tienen iguales letras afectadas de iguales exponentes Ejemplos: a) 4 xy , − 3xy , b)

1 xy 2

3 3 2 x y , − 2x 3 y 2 , 4

son términos semejantes

1 3x 3 y 2 , - x 3 y 2 2

son términos semejantes

Clasificación de las expresiones algebraicas A. Por su forma o naturaleza Se clasifican de acuerdo a la forma de sus exponentes que afectan a sus variables. a) Expresión algebraica racional (E.A.R.) Una expresión algebraica es racional cuando los exponentes de la parte literal( letras) son números enteros. Ejemplos: 4 y 2 − 5xy + 7z 4 ,

x 1 2 y −3 -3 +2 , , 5x + y (x + y) 2 7 xy 2

No son expresiones algebraicas racionales: 6 x 2 +

y ,

xy + 1 , 5x 1/2 + 3x 1/4 + 7 z 4


DAVID GONZÁLES LÓPEZ

- Expresión algebraica racional entera (E. A. R. E.).- Es aquella expresión algebraica racional que se caracteriza por presentar exponentes enteros positivos en su parte literal; es decir, no tiene parte literal en su denominador. Ejemplos: 4 x 2 y +

5x 3 − , 7x 3 − 6 x + 8 2 4

- Expresión algebraica racional fraccionaria ( E. A. R. F.).- Es aquella expresión algebraica racional que se caracteriza por presentar exponentes negativos en su parte literal; es decir, tiene parte literal en su denominador. Ejemplos:

4 xy , 7 x −8 + 6 x − 3 , 3 x+y x

b) Expresión algebraica irracional ( E. A. I. ) Una expresión algebraica es irracional cuando presenta exponentes fraccionarios en su parte literal. Ejemplos: 5x −1 / 3 + 8x −1 / 2 − 10 x −1 / 8 , 5x 9 − 6 x 8 + 4 x ,

12 3

x

+ 5 x7

Observa el siguiente esquema:

CLASIFICACIÓN DE LAS EXPRESIONES ALGEBRAICAS

Expresiones algebraicas racionales Enteras

Expresiones algebraicas irracionales

Fraccionarias

B. Por su número de términos Pueden ser: a) Monomio Es una expresión algebraica que consta de un sólo término. Dicho término es una expresión algebraica en la que las únicas operaciones que afectan a las letras son la multiplicación y la potenciación de exponente natural. También podemos decir, un monomio es una expresión algebraica racional entera que consta de un sólo término. Ejemplos: 4x , 7 x 5 y 3 y 5 x 4 y3z 2 b) Multinomio Es una expresión algebraica que consta de dos o más términos algebraicos Ejemplos: 3x 4 y + 2 x 3 y 4 − 7 x 2 y 5 y 4x 4 + 2 x −3 y 5 − 5 x + 3 Un caso particular de éstos es el polinomio o polinomio entero. Polinomio entero: es aquella expresión algebraica cuyos términos son todos expresiones algebraicas racionales enteras. Ejemplos: P(x) = 3x 4 + 2 x 3 − 5x + 4 y Q(x, y) = 3xy 3 + 7xy 4 + 5 x 4 z 8

5


DAVID GONZÁLES LÓPEZ

Grado de una expresión algebraica A. Grado de un monomio - Grado relativo: está dado por el exponente de la letra referida. - Grado absoluto: está dado por la suma de los exponentes de todas sus letras. Ejemplo: Sea el monomio: 12 x 6 y 3 z 9 Grado absoluto (GA ) : 6 + 3 + 9 = 18 Grado relativo (GR ) Grado relativo respecto a x (GR x ) es 6 Grado relativo respecto a y (GR y ) es 3 Grado relativo respecto a z (GR z ) es 9 B. Grado de un polinomio - Grado absoluto: está dado por el monomio de mayor grado absoluto. - Grado relativo: está dado por el mayor exponente de la letra referida. Ejemplo: Sea el polinomio: 3x 4 y 3 + 8x 3 y − 5 y Grado absoluto: Grado absoluto (GA ) de 3x 4 y 3 es 4 + 3 = 7 Grado absoluto (GA ) de 8x 3 y es 3 + 1 = 4 Grado absoluto (GA ) de es 5 y es 1 Por lo tanto, el Grado absoluto (GA ) del polinomio es 7 Grado relativo: Grado relativo respecto a x (GR x ) es 4 ( mayor exponente de la variable x ) Grado relativo respecto a y (GR y ) es 3 ( mayor exponente de la variable y ) Polinomios especiales A. Polinomio completo con respecto a una letra: es el polinomio que presenta todas las potencias de una letra, desde el mayor grado hasta el cero inclusive. El término algebraico que tiene la letra de grado cero se llama término independiente. Ejemplo: P(x, y) = 3x 3 y + 5x 2 − 4 xy 2 - 8 Este polinomio es completo respecto a x B. Polinomio ordenado con respecto a una letra: es el polinomio cuyos exponentes de la letra considerada, van aumentando o disminuyendo, según sea ascendente o descendente la ordenación. Ejemplo: P(x, y) = 8x 5 y + 6 x 3 y 4 − 7 xy 6 − 9 y 3 Este polinomio es ordenado con respecto a x C. Polinomio completo y ordenado con respecto a una letra: es aquel polinomio que presenta las dos características anteriores. Ejemplos:

P(x) = 8x 5 + x 4 + x 3 − 3x 2 − 4 x + 3 Q(x, y) = 5x 2 y 4 + xy 3 + y 2 − 3x 3 y + 4 ( polinomio completo y ordenado con respecto a y )

6


DAVID GONZÁLES LÓPEZ

D. Polinomio homogéneo: es el polinomio que presenta el mismo grado absoluto en todos sus términos. Ejemplo. P(x, y) = 7x 2 y 3 + 6 x 3 y 2 − 4 x 5 − 2 xy 4 E. Polinomios idénticos: son aquellos polinomios que presentan en sus términos semejantes, coeficientes iguales. Sea P( x ) = ax 3 + bx 2 + cx y Q( x ) = mx 3 + nx 2 + px Decimos que P( x ) ≡ Q( x ) ⇔ a = m , b = n , c = p Así : P( x ) = 3x 2 y −

8 x + 1 es idéntico a Q( x ) = 3x 2 y − 4 x + (4 − 3) 2

F. Polinomio nulo: es aquel polinomio que tiene todos sus coeficientes nulos o iguales a cero. También se le llama polinomio idénticamente nulo. Sea P( x ) = ax 3 + bx 2 + cx + d , decimos que P( x ) = 0 ⇔ a = 0 , b = 0 , c = 0 , d = 0 G. Polinomio opuesto: es aquel polinomio que se obtiene cambiando de signo a todos sus coeficientes del polinomio dado Sea P( x ) = ax 3 + bx 2 + cx + d decimos que − P( x ) es su opuesto ⇔ − P( x ) = −ax 3 − bx 2 − cx − d Así : Sea P( x ) = 2 x 3 + 3x 2 y − 4 x su opuesto es − P( x ) = −2 x 3 − 3x 2 y + 4 x Ejercicios 01: Grado de un polinomio y polinomios especiales 1. Hallar el grado absoluto de los siguientes polinomios: a) x 3 + x 2 + x c) x 3 y − x 2 y 2 + xy 4 − y 4 b) 5x − 3x 2 + 4 x 4 − 6 d) x 5 − 6 x 4 y 3 − 4 x 2 z + x 2 y 4 − 3y 6 2. Hallar el grado de los siguientes polinomios con relación a cada una de sus letras. a) x 3 + x 2 − xy 3 c) x 3 y 4 + x 2 z 3 + xy 5 z 2 b) x 4 + 4 x 3 − 6 x 2 y 4 − 4 xy 5

d) x 4 y 2 − xy 6 + 4 x 4 y 3 − x 8 + y 15 − z 11

3. Hallar a y b en el monomio 5x a − b y a + b si el grado relativo respecto a x es 8 y respecto a y es 12. 4. Hallar m y n sabiendo que el monomio : (m + n ) x 2 ( m −1) y 3n tiene grado absoluto igual a 17 y que además su coeficiente tiene el mismo valor que el grado relativo con respecto a x . 5. Si P( x , y, z ) = 7 x 2 y 5 z 8

6. Si Q( x , y, z ) = 5x 14 y 5 z 10

calcular

calcular

GR z + GA GR x + GR y

:

:

GR z + GA GR y + GR x

7. Sea el siguiente monomio M ( x , y) = 5x a y b

y sabiendo que GR x = 2 ∧ GA = 5

calcular a + b 8. Hallar el grado absoluto del siguiente polinomio : P( x ) = ( x + 5) 4 + (1 − x ) 5 2

2

9. Sea P( x , y) = x n + 2 y 6 − n + x n − 2 y n + 2 . Si cada término tiene el mismo grado absoluto, hallar n . 10. Sea P( x , y) = x a +1 y a −5 + x a −3 y a − 2 + x 2 a − 7 y + x a −5 y a , además GR x (P) = 7

7


DAVID GONZÁLES LÓPEZ

Hallar el GA( P) 11. Hallar (a + b) si P( x , y) = 3x a y a − b + 2 x a + b y a − b − 8x 8

es homogéneo.

12. Hallar (a − b − c + d ) si P( x ) = ( 2a − 12) x 3 + ( 4b + 8) x 2 + (c − 5) x + d a −18

a − b +15

es nulo.

c − b +16

13. Sea el polinomio P( x ) = 5x + 18x + 7x , hallar a , b y c para que el polinomio P( x ) sea completo y ordenado en forma descendente.

m n p x 2 − 10 x + 13 + + ≡ x − 1 x − 2 x − 3 ( x − 1)( x − 2)( x − 3) c + d −1 15. Se sabe que el polinomio P( x ) = 2 x + 5x b −c +1 + 7 x a + b− 4 + 8x a −3 es completo y ordenado descendentemente , calcular M = a + b + c + d . 16. Si el polinomio P( x ) = 3x p − n + 5 − 4 x n − m +3 + 7 x m − 6 es completo y ordenado ascendentemente, calcular el valor de ( 2m − 3n + 4p) . 14. Calcular E = m + n + p en la identidad

17. Determinar (a + b + c) sabiendo que el polinomio P( x ) = 3x 2 + ax − 5 + bx 2 − 11x + c es idénticamente nulo. 18. Si P( x ) es idénticamente nulo, hallar (a − b) 2 en P( x − a ) = a ( x + 3) + b( x + 2) + 2 . 19. Hallar E = ab(a + b) si el polinomio P( x , y) = x a − 2 b y a + b − 5x b y a + 2 b + 7 x a − b y 8 es homogéneo 20. En el polinomio homogéneo P( x , y) = ( x a y b ) a + b + ( x 3 y) ab el grado relativo a x es 48 . Indicar el valor de (a + b) . 21. Halle el número de términos del polinomio completo y ordenado

P( x ) = (n − 1) x n −6 + (n − 2) x n −5 + (n − 3) x n − 4 + ... 22. Si el polinomio P( x ) es ordenado y completo. Indicar cuántos términos tiene. P( x ) = (n − 3) x n −8 + (n − 4) x n −7 + (n − 5) x n −6 + ... Valor numérico de expresiones algebraicas Se denomina valor numérico de una expresión algebraica al resultado de sustituir cada una de las letras(variables) por números y realizar las operaciones indicadas. Ejemplos: 1. Hallar el valor numérico de E =

3x − y − 3 para x = −1 , y = -1/2 x+y Solución

1 1 5 3(−1) − (− ) −3+ − 2 −3 = 2 −3 = 2 −3 = −5 −3 = 5 −3 = 5−9 = − 4 E= 1 3 3 −3 3 3 3 (−1) + (− ) − − 2 2 2 2. Si P( x + 1) = x + 3 , hallar E = P( −3) + P( 4) Solución Primera forma: Haciendo x + 1 = k ⇒ x = k − 1 Escribiendo la expresión original en términos de k tenemos:

P(k ) = k − 1 + 3 ⇒ P(k) = k + 2 Una vez reducida se hace: k = x y se tiene P( x ) = x + 2

8


DAVID GONZÁLES LÓPEZ

hallamos P( −3) = −3 + 2 = −1 y P ( 4) = 4 + 2 = 6 Luego E = P( −3) + P( 4) = ( −3 + 2) + ( 4 + 2) = −1 + 6 = 5 Segunda forma: Reemplazamos x por x − 1 para hallar el P( x )

P( x + 1) = x + 3 P( ( x − 1) + 1 ) = ( x − 1) + 3 P( x ) = x + 2 hallamos P( −3) = −3 + 2 = −1 y P ( 4) = 4 + 2 = 6 Por lo tanto E = P( −3) + P( 4) = −1 + 6 = 5 Tercera forma: Escribiendo ( x + 3) en función de x + 1 :

P( x + 1) = ( x + 1) + 2 luego donde aparezca ( x + 1) se colocará x :

P( x ) = x + 2 Por lo tanto E = P( −3) + P( 4) = ( −3 + 2) + ( 4 + 2) = 5 Cuarta forma: Para calcular la expresión pedida podemos hacer:

x + 1 = −3 ⇒ x = −4 x +1 = 4 ⇒ x = 3 Estos valores se reemplazan en la igualdad original ( P( x + 1) = x + 3 ) P(−3) = −4 + 3 = −1 y P(4) = 3 + 3 = 6 Luego, E = P( −3) + P( 4) = −1 + 6 = 5

Ejercicios 02: Valor numérico de expresiones algebraicas Hallar el valor numérico de las siguientes expresiones algebraicas 1. E =

x 2 − y2 − z3 x 2 + y3

2. E = x 2 y 3 − 3x 3 y + 3

para

x = −2 , y = -1 , z = -1

para x = −1 , y = -1/2

3. E =

3x − 2 y + 1 2z − x + y

para x = −1 / 2 , y = 1/3 , z = -1/2

4. E =

x 3 − y2 + z3 x 2 − y3 − z 2

para x = −1 , y = 1/2 , z = -2

x 2 − 2y 2 z2 − z3 − para x = −1 , y = 1 , z = -2 z x 3xy xz 2 yz 6. E = + − + ( x + y − z) 2 − ( y + z − x )( x − y) z y x

5. E =

x + y x 2 − y2 7. E = − x y

para x = 1 , y = −1 , z = −2

para x = −2 , y = −1/2

9


DAVID GONZÁLES LÓPEZ

8. E = (

1 1 y − ) + ( − x) x y x

para x = −1 / 3 , y = −1/2

5x 2 − y 3 + 3y 9. E = para x = −1 / 2 , y = −2 y2 1− x y x2 − 3 − x 10. E = para x = −1 / 2 , y = −1 2 xy − x x - y 3 11. E = k (k − x )(k − y)(k − z) para x = 5 , y = −2 , z = 6 sabiendo que 3k = x + y + z z a+b+c 12. E = x ( x − a )( x − b)( x − c) donde x = , a = 2+ 2 , b = 2- 2 y c = 2 3 2 5x + 2 y 5x − 2 y x 4y 13. E = − si se cumple que + =2 x + 2y 3x + 2 y y x Resolver los siguientes ejercicios sobre valor numérico de un polinomio

P ( 0) + P ( − 2) P(−1) + P(−3) 2 15. Sea P( x ) = 4 x + 3x + 2 , calcular E = P( P(0)) + P( P( −1)) 16. Si P( x + 7) = x − 3 , calcular E = P( −1 / 2) + P(1 / 2) 14. Sea P( x ) = x 2 − x + 4 , calcular : E =

17. Sea P( x − 1) = 2 x − 5 , hallar P( 4) 18. Sabiendo que P( 2 x + 1) = x 2 + x + 1 y Q( x + 2) = x 2 − x + 1 , calcular P(Q( 2)) 19. Si tenemos que P( x 2 ) = x + 2 , Q(x 3 ) = x + 3 y R ( x 4 ) = x + 4 calcular E =

P(4) + Q(27) + R (16) 3x + 1 20. Si P( x ) = hallar P( P( x )) x −3 2x − 1 21. Si P( x ) = además P( P( x )) = 2 , hallar el valor de E = 5x + 6 x−2 mx + n m ) = x hallar P(−3) mx − n n 2x − 1 x 23. Si P(Q( x )) = y Q( x ) = , hallar P( x ) x x −1 24. Si P( x + a ) = x 2 − ax + b y Q( x − b) = x 2 − bx + a , determinar P(Q(0))

22. Si P(

25. Si P( x + 1) = 3x 2 − bx + 1 , Q(x - 1) = ax 2 + cx + 3 y P( x ) = Q( x + 1) , hallar E = a + b + c 26. Sabiendo que P( x + 1) + P( x + 2) = 5x y P(3) = 2 ,hallar E = P(5) − P( 2) 27. Se definen P (f ( x − 1) + 2) = x + 2 y P( x − 1) = x + 7 , a base de ello determinar f (7) 28. Si P( x ) = 4 x + 5 y P(g ( x ) + 3) = 8x + 5 , hallar g ( 4)

10


DAVID GONZÁLES LÓPEZ

1.3. OPERACIONES CON POLINOMIOS ENTEROS Adición y sustracción de polinomios La adición de polinomios es una operación que tiene por objeto reunir dos o mas polinomios (sumandos) en una sola expresión( suma) La sustracción de polinomios es la operación que consiste en sumar al polinomio minuendo el opuesto del polinomio sustraendo para obtener el polinomio diferencia. La adición y la sustracción de polinomios consiste en la reducción de términos semejantes Así, sean los polinomios P , Q , R y S P + Q = R y P − Q = P + ( −Q ) = S

Ejemplos 1. Hallar la suma de 2 x 2 − 3xy 2 + y 2 ; − 2 y 2 + 3xy 2 − 3x 2 ; 4x 2 + 7 xy 2 − y 2 − 3 Solución 2 2 2 2 2 Sea S = 2 x − 3xy + y + − 2 y + 3xy − 3x 2 + 4x 2 + 7 xy 2 − y 2 − 3 Agrupando términos semejantes y reduciéndolos se tiene:

S = (2 x 2 − 3x 2 + 4 x 2 ) + (−3xy 2 + 3xy 2 + 7 xy 2 ) + ( y 2 − 2 y 2 − y 2 ) − 3 S = 3x 2 + 7 xy 2 − 2 y 2 − 3 2. Sea P = 5x 3 + 2 x 2 y 2 − 2 y − 3 y Q = 2 x 3 − 5x 2 y 2 − 2 x − 3 hallar P − Q Solución

P − Q = P + (−Q) = 5x 3 + 2 x 2 y 2 − 2 y − 3 + −(2 x 3 − 5x 2 y 2 − 2 x − 3)

= 5x 3 + 2 x 2 y 2 − 2 y − 3 − 2 x 3 + 5x 2 y 2 + 2 x + 3 = (5x 3 − 2x 3 ) + (2x 2 y 2 + 5x 2 y 2 ) + (−3 + 3) − 2 y + 2x = 3x 3 + 7 x 2 y 2 − 2 y + 2 x 3. De x 2 y + y 2 − 5xy restar − 2 x 2 y + 3y 2 − 7 xy Solución 2 2 2 2 Sea M = x y + y − 5xy − ( −2 x y + 3y − 7 xy)

M = x 2 y + y 2 − 5xy + 2 x 2 y − 3y 2 + 7 xy M = ( x 2 y + 2 x 2 y) + ( y 2 − 3y 2 ) + (−5xy + 7 xy) M = 3x 2 y − 2 y 2 + 2 xy Ejercicios 03: Adición y sustracción de polinomios enteros Hallar la suma de : 1. x 2 − 3xy + y 2 ; - 2y 2 + 3xy − 3x 2 ; x 2 + 7 xy − y 2 2. x 3 + xy 2 + y 3 ; - 5x 2 y + x 3 − y 3 - 3x ; - 3x 3 − 4 xy 2 − 5 y 3 + x

11


DAVID GONZÁLES LÓPEZ

3. 4. 5. 6. 7. 8.

5 2 2 2 3 1 1 1 5 1 1 x − y + xy ; - xy − x 2 + y 2 ; xy − x 2 + y 2 6 3 4 2 6 8 6 3 4 2 3 3 5 3 x4 − x2 + 5 ; - x3 − x - 3 ; - x4 + x3 − x 3 8 5 6 4 2 5 3 1 1 5 1 1 x 4 − 2 x 2 y 2 + y 4 ; − x 4 + x 2 y 2 − xy 3 - y 4 ; − x 3 y − x 2 y 2 + y 4 7 6 8 6 14 6 4 7 2 4 3 1 2 1 1 x 5 − x 3 + x ; − 3x 5 + x 2 + x ; − x 4 + x 3 − x 2 3 5 8 10 3 6 4 2 3 5 2 1 3 3 2 7 2 1 3 2 3 1 2 1 a + ax − x ; − a x − ax − x ; − a + a x − ax 2 9 6 3 7 8 9 3 2 4 1 3 1 3 5 1 2 1 x 5 − y 5 ; − x 3 y 2 − xy 4 − y 5 ; − x 4 y − x 2 y 3 − x 5 ; 2x 4 y − x 3 y 2 − y 5 10 4 6 5 6 9 5 3

Resolver 9. De x 2 + y 2 − 3xy restar − y 2 + 3x 2 − 4 xy 10. De y 2 + 6 y 3 − 8 restar − 2 y 4 − 3y 2 + 6 y 11. De x 3 − 9 x + 6 x 2 − 19 restar − 11x 2 + 21x − 43 + 6 x 3 12. De y 5 − 9 y 3 + 6 y 2 − 31 restar − 11y 4 + 31y 3 − 8 y 2 − 19 y 13. Restar 6 x 3 − 9 x + 6 x 2 − 7 de x 4 − 8x 3 + 25x 2 + 15 14. Restar x 5 − x 2 y 3 + 6 xy 4 + 25 y 5 de − 3y 5 − 8x 3 y 2 − 19 x 5 − 15xy 4 15. Restar 23y 3 + 8 y 4 − 15 y 5 − 8 y − 5

3 2 7 3 9 xy − x + x 2 y − 8 9 2 3 2 17. Hallar la expresión que sumada con x − x + 5 da 3x − 6 18. Hallar la expresión que sumada con − 5a + 9b − x da 8x + 9a 1 3 1 3 3 3 1 5 2 19. De a − b restar la suma de − a 2 b + ab 2 con a 2 b − ab 2 + b 3 2 3 2 4 8 6 3 3 2 5 2 2 3 1 2 1 20. De la suma de x − xy + y con xy − y + restar la suma de 5 6 9 2 3 4 2 2 2 2 1 17 2 22 3 1 x − y + xy con x − xy − y 2 − 9 3 9 45 9 2 2 16. Restar −

1 2 3 x y + x3 + 6 6 4

de − 3y 5 − 8 y 3 + y 4 + 9

de

Multiplicación de polinomios La multiplicación de polinomios es la operación que consiste en obtener una expresión llamada producto, conociendo otras dos llamadas multiplicando y multiplicador Así, sean los polinomios P , Q y R

P⋅Q = R Propiedades : x m ⋅ x n = x m + n

y

( x m )n = x m.n

Ejemplos 1. Sea P = 3x 2 − 2 xy +

2 2 y 3

y

Q = 2 x − y hallar P ⋅ Q 12


DAVID GONZÁLES LÓPEZ

Solución

2 P.Q = (3x 2 − 2 xy + 6 y 2 )( x − y) 3 2 2 2 P.Q = (3x 2 )( x ) + (3x 2 )(− y) + (−2 xy)( x ) + (−2 xy)(− y) + (6 y 2 )( x ) + (6 y 2 )(− y) 3 3 3 4 P.Q = 2 x 3 − 3x 2 y − x 2 y + 2 xy 2 + 4 xy 2 − 6 y 3 3 4 P.Q = 2 x 3 + (−3x 2 y − x 2 y) + (2 xy 2 + 4 xy 2 ) − 6 y 3 3 13 P.Q = 2 x 3 − x 2 y + 6 xy 2 − 6 y 3 4 2. Sea P = 3x n − y 2

y

Q = 2x 2 n − x n y 2 + y 4

hallar P . Q Solución

P . Q = (3x n − y 2 ) (2 x 2 n − x n y 2 + y 4 ) = (3x n )(2 x 2 n ) + (3x n )(− x n y 2 ) + (3x n )( y 4 ) + (− y 2 )(2x 2 n ) + (− y 2 )(− x n y 2 ) + (− y 2 )( y 4 ) = 6 x 3 n − 3x 2 n y 2 + 3x n y 4 − 2 x 2 n y 2 + x n y 4 − y 6 = 6 x 3n + (−3x 2 n y 2 − 2 x 2 n y 2 ) + (3x n y 4 + x n y 4 ) − y 6 = 6 x 3n − 5x 2 n y 2 + 4 x n y 4 − y 6 Productos notables Son productos indicados que tienen una forma determinada, de los cuales se puede recordar fácilmente su desarrollo sin necesidad de efectuar la operación. Los más importantes son: 1. Binomio al cuadrado ( Trinomio cuadrado perfecto) • ( x + y)( x + y) = ( x + y) 2 = x 2 + 2 xy + y 2 •

( x − y)( x − y) = ( x − y) 2 = x 2 − 2 xy + y 2

(ax + by)(ax + by) = (ax + by) 2 = a 2 x 2 + 2abxy + b 2 y 2

(ax − by)(ax − by) = (ax − by) 2 = a 2 x 2 − 2abxy + b 2 y 2

2. Trinomio al cuadrado • ( x + y + z)( x + y + z) = ( x + y + z) 2 = x 2 + y 2 + z 2 + 2 xy + 2 xz + 2 yz •

(ax + by + cz)(ax + by + cz) = (ax + by + cz) 2 = a 2 x 2 + b 2 y 2 + c 2 z 2 + 2abxy + 2acxz + 2bcyz

3. Binomio al cubo • ( x + y)( x + y)( x + y) = ( x + y) 3 = x 3 + 3x 2 y + 3xy 2 + y 3 = x 3 + y 3 + 3xy( x + y) •

( x − y)( x − y)( x − y) = ( x − y) 3 = x 3 − 3x 2 y + 3xy 2 − y 3 = x 3 − y 3 − 3xy( x − y)

(ax + by)(ax + by)(ax + by) = (ax + by) 3 = a 3 x 3 + 3a 2 x 2 by + 3axb 2 y 2 + b 3 y 3

(ax − by)(ax − by)(ax − by) = (ax − by) 3 = a 3 x 3 − 3a 2 x 2 by + 3axb 2 y 2 − b 3 y 3

13


DAVID GONZÁLES LÓPEZ

4. Producto de una suma por su diferencia( Diferencia de cuadrados) • ( x m + y m )( x m − y m ) = ( x m ) 2 − ( y m ) 2 = x 2 m − y 2 m •

( x + y)( x − y) = x 2 − y 2

(ax + by)(ax − by) = (ax ) 2 − (bx ) 2 = a 2 x 2 − b 2 y 2

5. Producto de un binomio por un trinomio que da una suma o diferencia de cubos • ( x + y)( x 2 − xy + y 2 ) = x 3 + y 3 •

( x − y)( x 2 + xy + y 2 ) = x 3 − y 3

(ax + by)(a 2 x 2 − abxy + b 2 y 2 ) = (ax ) 3 + (by) 3 = a 3 x 3 + b 3 y 3

(ax − by)(a 2 x 2 + abxy + b 2 y 2 ) = (ax ) 3 − (by) 3 = a 3 x 3 − b 3 y 3

6. Identidad de ARGAND • ( x 2 m + x m y n + y 2 n )( x 2 m − x m y n + y 2 n ) = x 4 m + x 2 m y 2 n + y 4 n •

( x 2 k + x k + 1)( x 2 k − x k + 1) = x 4 k + x 2 k + 1

( x 2 + x + 1)( x 2 − x + 1) = x 4 + x 2 + 1

( x 2 + xy + y 2 )( x 2 − xy + y 2 ) = x 4 + x 2 y 2 + y 4

7. Producto de binomios con un término común • ( x + a )( x + b) = x 2 + (a + b) x + ab •

( x + a )( x + b)( x + c) = x 3 + (a + b + c) x 2 + (ab + ac + bc) x + abc

8. Producto de binomios de la forma (ax + b)(cx + d ) •

(ax + b)(cx + d ) = acx 2 + (ad + bc) x + bd

9. Identidades de Legendre • ( x + y ) 2 + ( x − y ) 2 = 2( x 2 + y 2 ) •

( x + y) 2 − ( x − y) 2 = 4 xy

(ax + by) 2 + (ax − by) 2 = 2(a 2 x 2 + b 2 y 2 )

(ax + by) 2 − (ax − by) 2 = 4abxy

10. Identidades de Lagrange • (ax + by) 2 + (ay − bx ) 2 = (a 2 + b 2 )( x 2 + y 2 ) •

(ax + by + cz) 2 + (ay − bx ) 2 + (az − cx ) 2 + (bz − cy) 2 = (a 2 + b 2 + c 2 )( x 2 + y 2 + z 2 )

Ejemplos

1. Simplificar E = [ ( x + y) 2 + ( x +

y)2 ( x − y)2

] (x 4 − x 2 y 2 + y 4 )

Solución Ordenado la expresión

E = { ( x + y) 2 + [ ( x + y ) ( x − y )

]2 }( x 4 − x 2 y 2 + y 4 )

aplicamos diferencia de cuadrados

E = [ ( x + y) 2 + ( x − y) 2

] (x 4 − x 2 y 2 + y 4 )

en el corchete aplicamos identidad de Legendre

14


DAVID GONZÁLES LÓPEZ

E = 2( x 2 + y 2 ) ( x 4 − x 2 y 2 + y 4 ) los paréntesis dan diferencia de cubos

E = 2( x 6 + y 6 ) Simplificar E = ( x 2 m + x m y n + y 2 n + x m − y n ) 2 − ( x 2 m + x m y n − x m + y 2 n + y n ) 2 + 4 y 3 n Solución Ordenado cada expresión

2.

E = [ (x 2m + x m y n + y 2n ) + (x m − y n )

haciendo x 2 m + x m y n + y 2 n = a

]2 − [ ( x 2m + x m y n + y 2 n ) − (x m − y n ) ]2 + 4 y 3n

y

x m − y n = b se tiene

E = (a + b ) 2 − ( a − b ) 2 + 4 y 3 n aplicando la identidad de Legendre

E = 4ab + 4 y 3n reemplazando los valores de a y b

E = 4( x 2 m + x m y n + y 2 n )( x m − y n ) + 4 y 3n de los paréntesis resulta diferencia de cubos

E = 4( x 3 m − y 3 n ) + 4 y 3 n E = 4 x 3 m − 4 y 3n + 4 y 3n E = 4 x 3m Ejercicios 04: Multiplicación de polinomios enteros- Productos notables Multiplicar 1. ( x 4 − 9)( x 4 − 7) 2. (m 2 + 1)(m 2 − 5)(m 2 − 4) 3. ( x + y + 3)( x − y + 4) 4. ( 2 x + 1)(4 x 2 − 2 x + 1)

1 1 1 2 3 xy + y 2 )( x − y) 2 3 4 3 2 2 1 1 3 6. ( x 2 + xy − y 2 )( x 2 + 2 y 2 − xy) 5 3 2 2 3 1 2 1 7. ( x 2 + x − )(2 x 3 − x + 2) 8 4 5 3 1 1 2 3 2 3 2 2 8. ( ax − x + a )( x − ax + a 2 ) 3 2 2 2 3 1 3 1 2 4 1 1 3 3 3 1 9. ( y + x y − x + x )( x y − x) 2 3 4 4 2 10 10. ( x n +1 + 2 x n + 2 − x n +3 )( x 2 + x ) 5. ( x 2 −

11. ( x n + 2 − 2 x n + 3x n +1 )( x n + x n +1 ) 12. ( x a + 2 − x a + 2 x a +1 )( x a + 3 − 2 x a +1 )

15


DAVID GONZÁLES LÓPEZ

13. (3x n −1 + x n − 2 x n − 2 )( x n − x n −1 + x n − 2 ) Simplificar 14. E = ( x + 1)( x − 1)( x 2 + 1)( x 4 + 1)( x 8 + 1) 15. E = ( y − k )( y + k )( y 2 + ky + k 2 )( y 2 − ky + k 2 ) 16. E = ( x + 1) 2 ( x − 2) 2 − ( x − 5) 2 ( x + 4) 2 − 36( x 2 − x ) 17. E = ( x + 2) 2 ( x − 1) 2 − ( x + 1) 2 ( x − 2) 2 18. E = ( x y + x − y )( x y − x − y )( x 4 y + 1 + x −4 y )

[

19. E = ( x y + y − y ) 2 + ( x y − y − y ) 2

] (x 2y − y −2 y )(x 8 y + x 4 y y −4 y + y −8 y )

20. E = ( 4 x + 1)( x + 1)( x + 1)( 4 x − 1)( x 4 + x 2 + 1)

[

]

21. E = ( 2 x + 3y) 2 + (2 y − 3x ) 2 ( y 2 − x 2 )( x 8 + x 4 y 4 + y 8 ) 22. E = ( x − y )( x + x y + y )( x 4 − x 2 y + y 2 ) 4

2

4

2

2

23. E = ( x 2 + xy + y 2 )( x − y )( x +

y)

24. E = ( x 3 − 1)( x + 1)( x 2 − x + 1)

[

25. E = (8m 3 − n 6 )(8m 3 + n 6 ) − ( 2m − n 2 )(2m + n 2 ) 2m( 2m + n 2 ) + n 4

] [ 4m

2

− n 2 ( 2m − n 2 )

]

26. E = ( x + y + z) + ( x + y − z) + ( x − y + z) + ( − x + y + z) − 4( x + y + z ) 2

2

2

2

2

2

2

1 2 2 1 1 1 ) ( x + 2 −1)2 − ( x − )2 ( x 2 + 2 + 1)2 x x x x 28. E = ( b + c − a ) (a + c - b) + (a + b + c)(a + b − c)

27. E = ( x +

29. E =

( x 2 − y 2 ) 7 (x + y) 4 ( y − x ) 5 (-x - y)10 (x - y) 2

30. E = ( x 2 + x + 1)( x 2 − x + 1)( x 4 − x 2 + 1)( x 8 − x 4 + 1) . . . hasta n factores Resolver 31. Si x + y = 8 y

xy = 16 , hallar x 2 + y 2

1 = 2 hallar x 4 + x −4 x 33. Si a + b = 5 y ab = 2 calcular E = (a 2 + b 2 ) + (a 3 + b 3 ) + (a 4 + b 4 )

32. Si x −

34. Si x + y + z = 12

y

xy + xz + yz = 60 , hallar M = ( x + y) 2 + ( x + z) 2 + ( y + z) 2

35. Si p + q + r = 20

y p 2 + q 2 + r 2 = 300 hallar el valor de E = ( p + q ) 2 + ( p + r ) 2 + (q + r ) 2

36. Si x + y + z = 0

simplificar E =

( x + y) 2 + ( y + z) 2 + ( z + x ) 2 xy + yz + zx

División de polinomios La división de polinomios es una operación que consiste en hallar el polinomio cociente dados el polinomio dividendo y el polinomio divisor. En la división de polinomios se cumple:

16


DAVID GONZÁLES LÓPEZ

P = Q⋅C + R

P R =C+ Q Q

ó

donde P = Polinomio dividendo Q = Polinomio divisor

C = Polinomio cociente R = Polinomio resto o residuo R C + = Cociente completo Q En la división exacta de polinomios R = 0 y se cumple que: P = C ⇔ P = Q⋅C donde Q ≠ 0 Q Propiedad :

xm = x m − n , donde x ≠ 0 n x

Casos de la división - Cuando se trata de dos monomios Reglas o pasos a seguir: • Se dividen los signos mediante la regla de signos • Se dividen los coeficientes •

Se dividen la letras aplicando la propiedad:

Ejemplo: Dividir E =

xm = x m − n , donde x ≠ 0 n x

− 12 x 4 y 8 z 5 − 4x 2 y 5 z 4 Efectuando

E = 3x 2 y 3 z - Cuando se trata de un polinomio y monomio Reglas o pasos a seguir: • Se escribe la división como una suma del cociente de 2 monomios • Se divide cada cociente de monomios Ejemplo: Dividir E =

− 4x 4 y 5 z 2 + 2 x 3 y 4 z 3 2 x 2 yz 2 Efectuando

E=

− 4x 4 y 5 z 2 2x 3 y 4 z 3 + = −2 x 2 y 4 + xy 3 z 2 2 2 2 2 x yz 2 x yz

- Cuando se trata de dos polinomios Para efectuar la división entre dos polinomios se conocen varios métodos. Presentaremos a continuación algunos de ellos. a. Método clásico o normal Reglas o pasos a seguir: 1) Se ordenan los polinomios, generalmente en forma decreciente con respecto a una sola letra o variable

17


DAVID GONZÁLES LÓPEZ

2) En caso existan dos o mas variables se asumirá solo a una de ellas como tal y las demás harán el papel de números o constantes. 3) Se divide el primer término del dividendo, por el primer término del divisor, obteniéndose el primer término del cociente. Luego este se multiplica por cada uno de los términos del divisor y el resultado se resta del dividendo. 4) Se baja el término siguiente del dividendo, y se repite el paso anterior tantas veces hasta que el resto sea a lo más un grado menos que el grado del divisor. O en todo caso si la división es exacta el resto será un polinomio idénticamente nulo. Ejemplos 1. Dividir P( x ) = 6 x 4 − 5x 3 − 4 x 2 + 8x + 4

entre Q( x ) = 2 x 2 − 3x + 2 Solución

6 x 4 − 5x 3 − 4 x 2 + 8 x + 4 − 6x 4 + 9x 3 − 6x 2 4 x 3 − 10 x 2 + 8x + 4 − 4x 3 + 6x 2 − 4x − 4x 2 + 4x + 4 4x 2 − 6 x + 4 − 2x + 8

2 x 2 − 3x + 2 3x 2 + 2 x − 2

Luego, el polinomio cociente es C( x ) = 3x 2 + 2 x − 2 y el resto R ( x ) = −2 x + 8 2. Dividir P( x , y) = 6 x 5 − 26 x 3 y 2 + 5x 4 y + 33x 2 y 3 − 24 xy 4 + 6 y 5 entre Q( x , y) = 2 x 2 − 3xy + y 2 Solución

6 x 5 + 5x 4 y − 26 x 3 y 2 + 33x 2 y 3 − 24 xy 4 + 6 y 5 − 6 x 5 + 9 x 4 y − 3x 3 y 2

2 x 2 − 3xy + y 2 3x 3 + 7 x 2 y − 4 xy 2 + 7 y 3

14 x 4 y − 29 x 3 y 2 + 33x 2 y 3 − 14 x 4 y + 21x 3 y 2 − 7 x 2 y 3 − 8x 3 y 2 + 26 x 2 y 3 − 24 xy 4 8x 3 y 2 − 12 x 2 y 3 + 4 xy 4 14 x 2 y 3 − 20 xy 4 + 6 y 5 − 14 x 2 y 3 + 21xy 4 − 7 y 5 xy 4 − y 5 Luego, el polinomio cociente es C( x ) = 3x 3 + 7 x 2 y − 4 xy 2 + 7 y 3 y el resto R ( x ) = xy 4 − y 5

18


DAVID GONZÁLES LÓPEZ

Observación - Teorema ( Algoritmo de la división de polinomios) Dados un polinomio P( x ) de grado n > 1 y un polinomio Q( x ) de grado m , con 1 ≤ m ≤ n ; R ( x ) , que tienen la propiedad de que: entonces existen polinomios únicos C( x ) y P( x ) = Q( x ) C( x ) + R ( x ) , donde el grado de R ( x ) es menor que el grado de Q( x ) . - Si al dividir P( x ) entre Q( x ) se obtiene R ( x ) = 0 , es decir si P( x ) = Q( x ) . C( x ) , se dice que Q( x ) divide o es divisor o factor de P( x ) . b. División sintética La división sintética es un procedimiento práctico para encontrar el cociente y el resto de la división de un polinomio P( x ) de grado 2 o mas, entre un binomio de la forma Q( x ) = x − r (o cualquier otra expresión transformable a ésta). A la división sintética también se le conoce con el nombre de “Regla de Ruffini” Si dividimos P( x ) = a n x n + a n −1 x n −1 + . . . + a 1 x + a 0

de grado n entre Q( x ) = x − r , entonces

por el algoritmo de la división de polinomios existe C( x ) = b n −1 x n −1 + b n − 2 x n − 2 + . . . + b1 de grado n − 1 y R ( x ) un polinomio constante talque : P( x ) = C( x )( x − r ) + R ( x ) Por división sintética podemos hallar el polinomio cociente C( x ) y el polinomio resto R ( x ). Reglas o pasos a seguir: - Se completan y ordenan los polinomios con respecto a una sola letra o variable. En caso falte un término este se completa con cero. - En caso hubiesen dos o mas variables se considera solo a una de ellas como tal y las demás harán el papel de números o constantes. Se distribuyen en forma horizontal los coeficientes del dividendo; en forma paralela a este paso se iguala el divisor a cero ( 0), se despeja la variable y ésta se coloca en el ángulo izquierdo del gráfico. - Se baja el primer coeficiente del polinomio dividendo siendo este el primero del polinomio divisor. Luego se multiplica por el valor despejado de la variable y el resultado se coloca debajo de la siguiente columna - Se reduce la columna siguiente y se repite el paso anterior tantas veces hasta que la última operación efectuada caiga debajo del último coeficiente del polinomio dividendo. Llegado este momento se reduce la columna que falta, y siempre se cumplirá que la última columna le va ha pertenecer al resto, y este siempre será un valor numérico. Observaciones - La división sintética también es aplicable cuando el divisor es un binomio de la forma ax − r . - La división sintética es también aplicable cuando el divisor es un polinomio de segundo grado factorizable de la forma ( x − r )( x − s) o no factorizable, o un polinomio de grado 2 o mas. Esta división se realiza por el llamado Método de Horner . Ejemplos 1. Dividir P( x ) = 2 x 3 − 5x 2 + x − 2 entre Q( x ) = x − 2 Solución Hacemos x − 2 = 0 ⇒ x = 2 Aplicando división sintética tenemos

19


DAVID GONZÁLES LÓPEZ

2

2 2

Luego , C( x ) = 2 x 2 − x − 1

−5 4 −1

−2 −2 −4

1 −2 −1

y R ( x ) = -4

2. Dividir P( x ) = 3x 4 + 2 x 3 − x 2 + 3 Hacemos x + 3 = 0 ⇒ x = −3 Aplicando división sintética tenemos

−3

entre Q( x ) = x + 3 Solución

3 3

Luego , C( x ) = 3x 3 − 7 x 2 + 20 x − 60

2 −9 −7

−1 0 21 − 60 20 − 60

3 180 183

y R ( x ) = 183

3. Dividir P( x ) = 18x 5 − 29 x 3 − 5x 2 + 12 x − 1 entre Q( x ) = 3x + 2 Solución

2 3

Hacemos Q( x ) = 3x + 2 = 3( x + )

2 ⇒ 3( x + ) = 0 3

x=−

2 3

Aplicando división sintética tenemos

2 3

18

18

0

− 29

−5

12

−1

− 12

8

14

−6

−4

− 12

− 21

9

6

−5

Cociente primario A ( x ) = 18x 4 − 12 x 3 − 21x 2 + 9 x + 6 Dividiendo todo el cociente primario entre 3 , porque es el primer coeficiente del divisor se tiene:

18x 4 − 12 x 3 − 21x 2 + 9 x + 6 3 C ( x ) = 6 x 4 − 4 x 3 − 7 x 2 + 3x + 2 y

El cociente verdadero C( x ) =

4. Dividir

R ( x ) = −5

6 x 36 + 17 x 27 − 16 x 18 + 2 x 9 + 12 3x 9 + 1

Solución Observamos que los exponentes del dividendo son múltiplos del exponente 9 del divisor, luego se puede aplicar el método de la división sintética. Hacemos x 9 = y . Al transformar el dividendo y reemplazar el cambio de variable se tiene

6( x 4 ) 9 + 17( x 9 ) 3 − 16( x 9 ) + 2 x 9 + 12 6 y 4 + 17 y 3 − 16 y 2 + 2 y + 12 = 3y + 1 3x 9 + 1

20


DAVID GONZÁLES LÓPEZ

1 3

También hacemos Q( y) = 3y + 1 = 3( y + )

1 ⇒ 3( y + ) = 0 3

y=−

1 3

Aplicando división sintética tenemos

1 3

6

17

6

−2 15

− 16

−5 − 21

2

12

7 9

−3 9

Cociente primario A ( y) = 6 y 3 + 15 y 2 − 21y + 9 Dividiendo todo el cociente primario entre 3 , porque es el primer coeficiente del divisor se tiene:

6 y 3 + 15 y 2 − 21y + 9 3 3 2 C( y ) = 2 y + 5 y − 7 y + 3

El cociente verdadero en términos de y , C( y) =

reemplazando y = x 9 tenemos el cociente verdadero en términos de x , C( x ) = 2 x 27 + 5x 18 − 7 x 9 + 3 y el resto R ( x ) = 9 c. Método de Horner Se emplea para dividir un polinomio P( x ) de grado n entre un polinomio Q( x ) de grado m donde P( x ) = a n x n + a n −1 x n −1 + . . . + a 1 x + a 0

Q( x ) = b m x m + b m −1 x m −1 + . . . + b1 x + b 0

,

n ≥ m , a n ∧ bm ≠ 0

Reglas o pasos a seguir: - Se completan y ordenan los polinomios. En caso falte un término este se completará con cero. - En caso existan dos o mas variables se asume a una de ellas como tal y las demás harán el papel de números o constantes. - Se distribuyen en forma horizontal los coeficientes del dividendo, y en forma vertical los coeficientes del divisor, todos cambiados de signo a excepción del primero. - Se divide el primer coeficiente del dividendo por el primero del divisor, obteniendo el primero del cociente. Luego este se multiplica por cada uno de los coeficientes del divisor que han cambiado de signo, y los resultados se colocan dejando una columna de lado. - Se reduce la siguiente columna y se repite el paso anterior, tanta veces hasta que la última operación efectuada caiga debajo del último coeficiente del dividendo. Llegado este momento se reduce las columnas que falten; separando los coeficientes del cociente y el resto respectivamente. Ejemplos 1. Dividir P( x ) = 2 x 4 + 6 x 3 + 4 x 2 − 5x − 1 entre Q( x ) = 2 x 2 + 2 x − 3

21


DAVID GONZÁLES LÓPEZ

Solución

2 −2 3

2

6 −2 4

1

Luego , C( x ) = x 2 + 2 x +

3 2

3

6

0 −1 2 2 Luego , C( x ) = 2 x 2 − 2

−5

−3

9 2

3 2

−2

7 2

2

y R ( x ) = −2 x +

2. Dividir P( x ) = 6 x 5 − 4 x 3 + 9 x 2 − 5

4 3 −4 3

−1

6

7 2

entre Q( x ) = 3x 3 + x − 2 Solución

0

−4

9

0

−5

0 0

−2 0 −6

4 0 0

2

−4

−2

13

2

−9

0

y R ( x ) = 13x 2 + 2 x − 9

3. Hallar E = m + n + p si la división

12 x 5 − 9 x 4 + 14 x 3 − mx 2 + nx − p 3x 3 + 2 x − 6

es exacta.

Solución Utilizando el método de Horner, el resto debe ser un polinomio idénticamente nulo

3

12

0 −2 6 4

−9

14

−m

n

−p

0 −9

−8 0 6

24 6 0

− 18 −4

12

− 22 + n

12 − p

−3

2

30 − m

Luego, el resto es C( x ) = (30 − m) x 2 + ( −22 + n ) + (12 − p) = 0 x 2 + 0 x + 0 Así tenemos que: 30 − m = 0 ⇒ m = 30

− 22 + n = 0 ⇒ n = 22 12 − p = 0 ⇒ p = 12 Finalmente E = m + n + p = 30 + 22 + 12 = 64

22


DAVID GONZÁLES LÓPEZ

8x 5 + 14 x 4 y + 5x 3 y 2 + 16 x 2 y 3 + 2 y 5 4 x 2 + xy + 3y 2

3. Dividir

con respecto a x .

Solución Utilizando el método de Horner se tiene:

4 −1 −3

8

14

5

−2

−6 −3 −4

12

2

3

−1

16

0

−9 1 8

3 −2

2

1

2

−6 −4

Luego, el resto es C( x , y) = 2 x 3 + 3x 2 y − xy 2 + 2 y 3 y R ( x , y) = xy 4 − 4 y 5 Teorema del resto Si P( x ) es un polinomio de grado n y Q( x ) = x − r , entonces el resto o residuo de dividir P( x ) por Q( x ) esta dado por P(r ) = R . Demostración En efecto, por el algoritmo de la división

P ( x ) = ( x − r ) C( x ) + R

Como esta igualdad es válida para todo x , en particular para x = r entonces

P ( r ) = ( r − r ) C( r ) + R Entonces P( r ) = R

Ejemplos 1. Hallar el resto de dividir P( x ) = x 3 + x 2 − 4 x − 1 entre Q( x ) = x + 2 Solución Hacemos x + 2 = 0 ⇒ x = −2 Luego el resto es R = P( −2) = ( −2) 3 + ( −2) 2 − 4( −2) − 1 = 3

2. Hallar el resto de

9 x 3 − 18x 2 + x − 2 3x − 2 Solución

2 Hacemos 3x − 2 = 0 ⇒ x= 3 2 2 3 2 2 20 Luego el resto es R = P( ) = 9( ) − 18( ) 2 + − 2 = − 3 3 3 3 3 3. Hallar el resto de

x 8 − 2 x 5 − 3x 4 − 8 x 2 − 5 x + 8 x3 + 2

Hacemos x + 2 = 0 ⇒ Dando la forma al dividendo 3

x = −2 3

Solución ¡ no se saca raíz !

23


DAVID GONZÁLES LÓPEZ

P( x ) = ( x 3 ) 2 x 2 − 2( x 3 ) x 2 − 3( x 3 ) x − 8x 2 − 5x + 8 reemplazando

R = (−2) 2 x 2 − 2(−2) x 2 − 3(−2) x − 8x 2 − 5x + 8 efectuando resulta

R = x +8 4. Hallar el resto de

( x 2 + 5x + 1) 3 − 3( x 2 + 5x + 2) 2 + 4 x 2 + 5x − 1 Solución

Hacemos que x + 5x − 1 = 0 Reemplazando en el dividendo 2

x + 5x = 1 2

R = (1 + 1) 3 − 3(1 + 2) 2 + 4 = 8 − 27 + 4 R = −15 5. Hallar el resto de

( x + y) 5 − x 5 − y 5 x + 2y Solución

⇒ Hacemos que x + 2 y = 0 Reemplazando en el dividendo

x = −2 y

R = (−2 y + y) 5 − (−2 y) 5 − y 5 = − y 5 + 32 y 5 − y 5 R = 30 y 5 Teorema del factor Si P( x ) es un polinomio de grado n , entonces un número r es una raíz de P( x ) si y solo si Q( x ) = x − r es un factor de P( x ) . Demostración i) En efecto, por el algoritmo de la división

P ( x ) = ( x − r ) C( x ) + R Por el teorema del resto P( r ) = 0 , entonces R = 0 Por lo tanto, P( x ) = ( x − r ) C( x ) , luego x − r es un factor de P( x ) ii) Recíprocamente, si Q( x ) = x − r es un factor de P( x ) , entonces P( x ) = ( x − r )C( x ) Como el resto R = P( r ) = 0 , entonces P( r ) = ( r − r ) C( r ) = 0 Significa que r es una raíz de P( x ) . Ejemplos 1. Determinar si Q( x ) = x + 2 es factor de P( x ) = x 3 − x 2 + 5x + 4 Solución Hacemos x + 2 = 0 ⇒ x = −2 3 Entonces R = P( −2) = ( −2) − ( −2) 2 + 5( −2) + 4 = −18 Luego Q( x ) = x + 2 no es factor de P( x ) = x 3 − x 2 + 5x + 4

24


DAVID GONZÁLES LÓPEZ

2. Determinar si Q( x ) = 2 x − 1 es factor de P( x ) = 4 x 3 + 12 x 2 − x − 3 Solución Hacemos 2 x − 1 = 0

x=

1 2

1 2

1 1 1 2 2 2 Como R = 0 entonces Q( x ) = 2 x − 1 es factor de P( x ) = 4 x 3 + 12 x 2 − x − 3 Entonces R = P( ) = 4( ) 3 + 12( ) 2 − ( ) − 3 = 0

Observación - Raíces de un polinomio: De acuerdo al teorema del factor se conoce que dado un polinomio de grado n ≥ 1 , un número r se llama raíz o cero del polinomio P( x ) si P( r ) = 0 . Ejemplo: Sea P( x ) : x 3 − x 2 − 4 x + 4 , el número x = −2 es una raíz o un cero de P( x ) puesto que P( −2) = 0 Cocientes notables Son divisiones indicadas de dos expresiones binómicas. Se denominan notables porque no se requiere efectuar la operación, directamente se escribe el cociente. - Primer caso

xn − an = x n −1 + ax n − 2 + a 2 x n −3 + . . . + a n − 2 x + a n −1 x −a

donde: n es par o impar Ejemplos *

x 5 − y5 = x 4 + x 3 y + x 2 y 2 + xy 3 + y 4 x−y

*

x 8 − y 8 (x 2 ) 4 − ( y 2 ) 4 = = ( x 2 ) 3 + ( x 2 ) 2 ( y 2 ) + ( x 2 )( y 2 ) 2 + ( y 2 ) 3 2 2 2 2 x −y x −y = x 6 + x 4 y2 + x 2 y4 + y6

- Segundo caso

xn − an = x n −1 − ax n − 2 + a 2 x n −3 − . . . + a n − 2 x + a n −1 x+a

donde n es par Ejemplo *

x 8 − y 20 ( x 2 ) 4 − ( y 5 ) 4 = = ( x 2 ) 3 − ( x 2 ) 2 ( y 5 ) + ( x 2 )( y 5 ) 2 − ( y 5 ) 3 2 5 2 5 x +y x +y = x 6 − x 4 y 5 + x 2 y 5 + x 2 y10 − y15

- Tercer caso

xn + an = x n −1 − ax n − 2 + a 2 x n −3 − . . . − a n − 2 x + a n −1 x+a

donde n es impar *

x 10 + y10 ( x 2 ) 5 + ( y 2 ) 5 = = ( x 2 ) 4 − ( x 2 ) 3 ( y 2 ) + ( x 2 ) 2 ( y 2 ) 2 − ( x 2 )( y 2 ) 3 + ( y 2 ) 4 2 2 2 2 x +y x +y = x 8 − x 6 y 2 + x 4 y 4 − x 2 y 6 + y8

25


DAVID GONZÁLES LÓPEZ

xn + an x−a donde n es par o impar

- Caso

Por el Teorema del resto: x − a = 0 ⇒ x = a Luego, R = a n + a n = 2a n ¡ División inexacta! , por lo tanto NO ES COCIENTE NOTABLE. - Caso

xn − an x+a

donde n es impar Por el Teorema del resto: x + a = 0 ⇒ x = −a Luego, R = ( −a ) n − a n = −2a n ¡ División inexacta! , por lo tanto NO ES COCIENTE NOTABLE.

xn + an x+a donde n es par

- Caso

Por el teorema del resto x + a = 0 ⇒ x = −a Luego, R = ( −a ) n + a n = a n + a n = −2a n ¡ División inexacta! , por lo tanto NO ES COCIENTE NOTABLE Ejercicios 05: División de polinomios enteros- Cocientes notables Dividir 1. 3x 2 y 3 − 5x 2 y

entre − 3x 2 y

2. 4 x 8 − 10 x 6 − 8x 4 entre 2 x 3 3. 21x 18 y18 − 35x 12 y 9 z 2 − 7 x 8 y10 z 8 entre − 7 x 5 y 4. 5. 6. 7. 8.

1 4 2 3 3 1 m − m n + m 2 n 2 entre m 2 4 3 8 4 2 4 3 1 3 4 1 2 5 1 x y − x y + x y − xy 6 entre − xy 3 3 5 4 5 3 3 2 5 1 5 x yz − x 2 y 2 z 2 + xyz − x 4 yz 3 entre − xyz 4 3 6 2 6 m+2 m m +1 m −1 m−2 x − 15x + 6 x − 9 x entre 3x 2 n +1 1 n −1 2 n 2 n−2 x − x − x entre x 3 4 5 3

Dividir por el método clásico y por división sintética 9. x 3 − 2 x 2 + 3x − 2 entre x − 3 10. 2 x 4 − 3x 2 + 3x − 1 entre x − 2

3 3 2 2 5 1 1 entre x − x − x + x− 4 3 6 2 2 4 3 12. 4 x − 3x + x − 2 entre 2 x + 1 11.

26


DAVID GONZÁLES LÓPEZ

Dividir por el método clásico y por el método de Horner 13. x 4 − 3x 3 + x 2 − x + 1 entre x 2 − x + 2 14. 2 x 5 − 2 x 3 + 3x 2 + 1 entre 2 x 2 − x − 2

1 4 2 3 x − x − x 2 + x − 1 entre 3x 2 + 2 3 3 5 16. x − 3x 4 − x 3 + x − 2 entre 2 x 3 − x 2 + 3x − 1 15.

Resolver

x 4 + 2 x 3 − 3x 2 + ax − b 17. Calcular ab si la siguiente división es exacta x 2 + 2x − 5 x 4 − 3x 3 + mx − 2n 18. Hallar m y n si la división es exacta x 2 − 2x + 4 5x 4 − 11x 3 + 15x 2 + ax − b 19. Calcular a + b si la división deja como resto: 2 x − 9 5x 2 − x − 2 20. Calcular a.b.c si el polinomio x 4 + 3x 3 + ax 2 + bx + c es divisible por x 3 + 2 x 2 − x − 2 x 4 + 2 x 3 − m 2 x 2 + mx − m 21. Hallar el resto en x − m +1 80 77 x + 8x + 2 x + 5 22. Hallar el resto en x+2 27 x + 243x 22 + x + 4 23. Hallar el resto en x+3 5 8x + 4 x 3 + ax 2 + bx + c 24. El residuo de dividir es : 5x 2 + 11x + 7 , hallar E = abc 3 2 2x + x + 3 25. Determinar m + n sabiendo que mx 4 + nx 3 − 7 x 2 + 16 x + 15 es divisible por x 2 − 3x + 5 3x 4 − x 3 + 2 x 2 + ax + a 26. En la siguiente división el residuo no es de primer grado. Calcular x2 + x −1 dicho resto. 27. Si P( x ) = x 3 − bx 2 − ( 4b + 3) x + c es divisible entre ( x + 3)( x − 4) , hallar P(1) 28. Escribir el cociente sin efectuar la división :

81x 4 − 16 y 4 a) 3x − 2 y b)

x 5 − 243 x −3

1024 x 10 − 1 c) 2x − 1 d)

16 x 2 y 4 − 25m 6 4 xy 2 + 5m 3

32 x 5 + 243y 5 e) 2 x + 3y f)

64 x 6 − 343y 9 4x 2 − 7 y 3

512 x 9 + y 9 g) 2x + y h)

a 18 − b18 a 3 + b3

1.4. FACTORIZACIÖN Factorizar es la transformación de un polinomio en una multiplicación indicada de dos o mas polinomios primos dentro de cierto campo de números Factorizar también significa, convertir una suma algebraica en producto de factores primos.

27


DAVID GONZÁLES LÓPEZ

Polinomio primo sobre un conjunto numérico Es aquel polinomio de grado mayor que cero que no admite ser transformado en multiplicación indicada. Todo polinomio primo tiene como únicos divisores a el mismo y a cualquier constante no nula de un cierto campo de números. Un polinomio primo también es denominado polinomio irreductible y factor primo. Ejemplos 1. 3x + 2 es primo en Q, R y C 2. x 2 − 5

es primo en Q . No es primo en R

3. x 2 − x + 1 es primo en Q y R . No es primo en C Generalmente trabajaremos en los racionales (Q) salvo que se indique lo contrario. Pero debe tenerse en cuenta lo siguiente: * P( x ) = ( x 2 + 1)( x 2 − 3)( x + 4)( x − 2) está factorizado en Q . * P( x ) = ( x 2 + 1)( x + 3 )( x − 3 )( x + 4)( x − 2)

está factorizado en R .

* P( x ) = ( x + i)( x − i)( x + 3 )( x − 3 )( x + 4)( x − 2)

está factorizado en C .

El número de factores primos, como lo hemos visto anteriormente depende del conjunto numérico en el que se trabaje. En el conjunto numérico de los racionales, el número de factores primos se calcula contando los factores basales (que figuran como bases y que contengan a las variables, denominados también factores algebraicos). Así por ejemplo: * P( x ) = ( x + 3) 2 ( x − 4) 3 tiene 2 factores primos * P( x ) = x ( x + 2)( x − 2) 2 ( x − 3) 3 tiene 4 factores primos * P( x ) = x 2 y 2 ( x + 3y) 5 ( x − 2 y) 4 tiene 4 factores primos Métodos para factorizar una expresión algebraica A. Método de factor común B. Método de identidades C. Método del aspa simple D. Regla de Ruffini E. Método de los artificios: a) Cambio de variable o agrupaciones convenientes b) Quita y pon o reducción a diferencia de cuadrados c) Sumas y restas especiales

A. Método de factor común Factor común de dos o más expresiones algebraicas es la parte numérica y/o literal que está repetida en dichas expresiones. El método consiste en extraer el o los factores comunes y dejarlos como un producto indicado. El factor común puede ser: • Factor común MONOMIO • Factor común POLINOMIO • Factor común POR AGRUPACIÖN DE TËRMINOS Ejemplo: 1. Factorizar la siguiente expresión: P = 3x 2 y 2 + 6 x 3 y 5 + 12 x 2 y 4

28


DAVID GONZÁLES LÓPEZ

Solución El factor común es 3x 2 y 2 , es un monomio

P = 3x 2 y 2 (1 + 2 xy 3 + 4 y 2 ) 2. Factorizar la siguiente expresión: P = 72 x 2 a y b + 48x a +1 y b +1 + 24 x a y 2 b Solución El factor común es 24 x a y b , es un monomio

P = 24 x a y b (3x a + 2 xy + y b ) 3. Factorizar la siguiente expresión: P = ( x + 1) 7 ( x 2 + 1)10 − ( x + 1) 5 ( x 2 + 1)11 Solución 5 2 10 El factor común es ( x + 1) ( x + 1) , es un polinomio

[

P = ( x + 1) 5 ( x 2 + 1)10 (x + 1) 2 − ( x 2 + 1)

]

P = ( x + 1) ( x + 1) ( x + 2 x + 1 − x − 1) 5

2

10

2

2

P = ( x + 1) 5 ( x 2 + 1)10 ( 2 x ) 4. Factorizar N = a ( x − 1) − b(1 − x ) + cx − c Solución Extrayendo factor común " c" en los dos últimos términos

N = a ( x − 1) − b(1 − x ) + c( x − 1) Extrayendo factor común − 1 en el término central N = a ( x − 1) + b( x − 1) + c( x − 1) Extrayendo factor común ( x − 1) N = ( x − 1)(a + b + c) 5. Factorizar M = a 3 x 3 + a 2 x 2 b + a 2 x 2 c + a 2 x 2 d + abcx + abdx + acdx + bcd Solución Agrupando de dos en dos

M = (a 3 x 3 + a 2 x 2 b) + (a 2 x 2 c + abcx ) + (a 2 x 2 d + abdx ) + (acdx + bcd) Extrayendo factor común en cada paréntesis

M = a 2 x 2 (ax + b) + acx (ax + b) + adx (ax + b) + cd(ax + b) Extrayendo factor común (ax + b) M = (ax + b)(a 2 x 2 + acx + adx + cd) Agrupando de dos en dos en el segundo paréntesis

[

M = (ax + b) (a 2 x 2 + acx ) + (adx + cd)

]

Extrayendo factor común dentro del corchete

M = (ax + b)[ax (ax + c) + d(ax + c)] M = (ax + b)(ax + c)(ax + d )

29


DAVID GONZÁLES LÓPEZ

B. Método de identidades Para este caso se utilizará los productos notables en forma inversa; entre los mas importantes tenemos: • Trinomio cuadrado perfecto. Se caracteriza por: - Tener dos términos que son cuadrados perfectos. - El otro término es el doble producto de las raíces cuadradas de los cuadrados perfectos. - Los cuadrados perfectos siempre deben tener signo positivo. El trinomio con estas características se reduce a un binomio al cuadrado. Para factorizarlo se extrae la raíz del primer y tercer término y entre ellas va el signo del segundo término. Forma general : a) x 2 m + 2 x m y n + y 2 n = ( x m + y n ) 2 b) x 2 m − 2 x m y n + y 2 n = ( x m − y n ) 2 Ejemplos: 1. x 2 + xy + y 2 = ( x + y) 2 2. 4 x 4 y 6 − 4 3 x 3 y 3 + 3x 2 = ( 2 x 2 y 3 − 3 x ) 2 • Diferencia de cuadrados. Es una diferencia de dos cuadrados perfectos. Para factorizar se extrae la raíz cuadrada de los cuadrados perfectos y se forma el producto de la suma de las raíces multiplicada por la diferencia de ellas. Forma general : x 2 m − y 2 n = ( x m + y n )( x m − y n ) Ejemplos: 1. x 2 − y 2 = ( x − y)( x + y) 2. 9 x 4 y 2 − 5 y 8 = (3x 2 y + 5 y 4 )(3x 2 y − 5 y 4 ) • Suma o diferencia de cubos Se caracterizan por tener 2 cubos perfectos, para factorizar se recuerda el producto notable Forma general a) x 3m + y 3 n = ( x m + y n ) ( x 2 m − x m y n + y 2 n ) b) x 3m − y 3n = ( x m − y n ) ( x 2 m + x m y n + y 2 n ) Ejemplos: 1. x 3 + y 3 = ( x + y) ( x 2 − xy + y 2 ) 2. 8x 3 z 6 − 27 y 9 = ( 2 xz 2 − 3y 3 )(4 x 2 z 4 + 6 xy 3 z 2 + 9 y 6 ) Otros ejemplos 1. Factorizar P = m 2 − 4p 2 + 4mn + 4n 2 Solución Agrupando el 1º , 3º y 4º término

P = (m 2 + 4mn + 4n 2 ) − 4p 2 Factorizando el paréntesis

P = (m + 2n ) 2 − 4p 2 Factorizando toda la expresión

P = ( m + 2 n + 2 p) ( m + 2n − 2 p )

2. Factorizar P = a 4 + b 4 + 2ab(a 2 + b 2 ) + 3a 2 b 2

30


DAVID GONZÁLES LÓPEZ

Sabemos que: 3a b = 2a b + a b 2

2

2

2

2

2

Solución , luego

P = a 4 + b 4 + 2ab(a 2 + b 2 ) + 2a 2 b 2 + a 2 b 2 agrupando adecuadamente

P = (a 4 + 2a 2 b 2 + b 4 ) + 2ab(a 2 + b 2 ) + a 2 b 2 Factorizando el trinomio cuadrado perfecto

P = (a 2 + b 2 ) 2 + 2ab(a 2 + b 2 ) + a 2 b 2 Toda la expresión es un trinomio cuadrado perfecto

P = [ (a 2 + b 2 ) + ab

]2

P = (a 2 + b 2 + ab) 2 3. Factorizar E = x ( x 2 + yz) + z( x 2 + y 2 ) − y 3 Solución Efectuando las operaciones indicadas

E = x 3 + xyz + x 2 z + y 2 z − y 3 Agrupando adecuadamente

E = ( x 3 − y 3 ) + ( xyz + x 2 z + y 2 z) Factorizando los paréntesis

E = ( x − y) ( x 2 + xy + y 2 ) + z( xy + x 2 + y 2 ) El factor común es ( x 2 + xy + y 2 )

E = ( x 2 + xy + y 2 )( x − y + z) 4. Factorizar P = x 3 + x 2 + x − 3 Solución Sabemos que: − 3 = −1 − 1 − 1 , luego

P = x3 + x2 + x −1−1−1 Agrupando adecuadamente

P = ( x 3 − 1) + ( x 2 − 1) + ( x − 1) Factorizando en el 1º y 2º paréntesis

P = ( x − 1)( x 2 + x + 1) + ( x − 1)( x + 1) + ( x − 1) El factor común es: ( x − 1) P = ( x − 1)( x 2 + x + 1 + x + 1 + 1) P = ( x − 1)( x 2 + 2 x + 3) C. Método del aspa simple Se utiliza para factorizar trinomio de la forma: a) x 2 n ± bx n ± c ó b) ax 2 n ± bx n ± c Para factorizar se hace lo siguiente: - Se descompone en dos factores el primer término, estos factores se colocan en las puntas de la izquierda del aspa. - Se descomponen en dos factores el término independiente, incluyendo el signo, estos factores se colocan en las puntas de la derecha del aspa. - El término central debe ser igual a la suma de los productos del aspa.

31


DAVID GONZÁLES LÓPEZ

- Los factores son las sumas en forma horizontal de los extremos del aspa. Ejemplos 1. Factorizar F = 8x 2 + 14 x − 15 Solución

F = 8x 2 + 14 x − 15 4x −3 2x 5

⇒ − 6x ⇒ 20 x 14 x La expresión factorizada es: F = 8x 2 + 14 x − 15 = ( 4 x − 3)(2 x + 5)

2. Factorizar F = 25x 4 − 109 x 2 + 36 Solución

F = 25x 4 − 109 x 2 + 36 25x 2 −9 ⇒ 2 x −4 ⇒

− 9x 2 − 100 x 2

− 109 x 2 La expresión factorizada es: F = 25x 4 − 109 x 2 + 36 = ( 25x 2 − 9)( x 2 − 4)

F = 25x 4 − 109 x 2 + 36 = (5x − 3)(5x + 3)( x − 2)( x + 2) 3. Factorizar F = 64 x 12 y 3 − 68x 8 y 7 + 4 x 4 y11 Solución El factor común de la expresión es 4 x 4 y 3

F = 4 x 4 y 3 (16 x 8 − 17 x 4 y 4 + y 8 ) Aplicamos aspa simple en el paréntesis y tenemos

F = 4 x 4 y 3 (16 x 4 − y 4 ) ( x 4 − y 4 ) Factorizando las diferencias de cuadrados tenemos

F = 4 x 4 y 3 (4 x 2 − y 2 )(4 x 2 + y 2 ) ( x 2 − y 2 )( x 2 + y 2 ) Factorizando las diferencias de cuadrados en el primer y tercer paréntesis

F = 4 x 4 y 3 (2 x − y)(2 x + y)(4 x 2 + y 2 ) ( x − y)( x + y)( x 2 + y 2 ) 4. Factorizar M = (a + b) 2 + (c + 4)(a + b) − 2c 2 + 5c + 3 Solución Extrayendo el signo menos en los tres últimos términos

M = (a + b) 2 + (c + 4)(a + b) − (2c 2 − 5c − 3) Aplicando aspa simple en el último paréntesis

M = (a + b) 2 + (c + 4)(a + b) − (2c + 1)(c − 3) Aplicando aspa simple en toda la expresión

M = (a + b) 2 + (c + 4)(a + b) − (2c + 1)(c − 3) (a + b ) (2c + 1) ⇒ (a + b)(2c + 1) 32


DAVID GONZÁLES LÓPEZ

(a + b )

− (c − 3)

(a + b)(−(c − 3)) (c + 4)(a + b)

La expresión factorizada es:

M = (a + b) 2 + (c + 4)(a + b) − 2c 2 + 5c + 3 = (a + b + 2c + 1)(a + b − c + 3) Observación Recordemos que: - Sea P( x ) un polinomio de grado n (n ≥ 1) , " r" es raíz o cero de P( x ) ⇔ P( r ) = 0 Es decir, raíz o cero es el valor que anula al polinomio. D. Regla de Ruffini La regla de Ruffini se basa en el siguiente teorema: Sea P( x ) = a n x n + a n −1 x n −1 + . . . + a 1 x + a 0 , an ≠ 0 y

a 0 ≠ 0 un polinomio de grado n

p , expresado en forma irreductible, es q una raíz de P( x ) , entonces p es divisor exacto de a 0 y q es divisor exacto de a n . cuyos coeficientes son enteros. Si el número racional

La Regla de Ruffini permite factorizar polinomios de grado 2 o más que acepte factores de primer grado de la forma x ± r ó sx ± r . También, podemos decir que la Regla de Ruffini sirve para hallar los divisores binómicos de un polinomio. Pasos a seguir - Determinación de las posibles raíces racionales ( o ceros ) de un polinomio de grado 2 o más: *Si el polinomio tiene como primer coeficiente la unidad, las posibles raíces o ceros estarán dados por los divisores del término independiente con su doble signo. *Si el coeficiente del primer término es diferente a la unidad, se procede como en el caso anterior y además se consideran las fracciones que resultan de dividir todos los divisores del término independiente entre los divisores del primer coeficiente. Así: Posibles raíces racionales de un polinomio ( PRR) Dado: P( x ) = a n x n + a n −1 x n −1 + . . . + a 1 x + a 0 , donde a n ≠ 0 , a 0 ≠ 0 y n ∈ z +

 Divisores positivos de a 0    Divisores positivos de a n 

PRR = ± 

- Luego, se utiliza la Regla de Ruffini ( o división sintética) tantas veces como ceros o raíces tenga el polinomio. Ejemplos: 1. Factorizar P( x ) = x 3 − x 2 − 4 x + 4

33


DAVID GONZÁLES LÓPEZ

Solución Posibles raíces o ceros: PRR = ±1 , ± 2 , ± 4

1

1

2

1

−2

1

−1 1 0 2 2 −2 0

1

−4 4 0 −4 0 −4 4 0

Luego, el polinomio factorizado es igual a: P( x ) = x 3 − x 2 − 4 x + 4 = ( x − 1)( x − 2)( x + 2) 2. Factorizar P( x ) = x 4 + 3x 3 − x 2 − 5x + 2 Solución Posibles raíces o ceros: PRR = ±1 , ± 2

1

1

−2

1

−1 4 3 −4 −1

3 1 4 −2 2

1

−5 3 −2 2 0

2 −2 0

El polinomio factorizado es igual a: P( x ) = x 4 + 3x 3 − x 2 − 5x + 2 = ( x − 1)( x + 2)( x 2 + 2 x − 1) 3. Factorizar P( x ) = 12 x 3 + 20 x 2 + x − 3 Solución

±1, ± 3    ± 1 , ± 2 , ± 3 , ± 4 , ± 6 , ± 12 

Posibles raíces o ceros: PRR = 

PRR = ± 1 , ±

1 1 1 1 1 3 3 , ± , ± , ± , ± , ± 3, ± , ±  2 3 4 6 12 2 4

1 2

12

20

1

−3

1 3

12

−6 14

−7 −6

3 0

3 2

12

4 18

6 0

12

− 18 0

34


DAVID GONZÁLES LÓPEZ

1 2

1 3

3 2

El polinomio factorizado es igual a P( x ) = 12 x 3 + 20 x 2 + x − 3 = 12( x + )( x − )( x + ) También llegamos a factorizarlo de la siguiente manera:

1 2

Como ( x + ) es factor de P( x ) Tenemos P( x ) = (

2x + 1 2x + 1 ) (12 x 2 + 14 x − 6) = ( ) 2(6 x 2 + 7 x − 3) = (2 x + 1)(3x − 1)(2 x + 3) 2 2

4. Factorizar P( x ) = 2 x 5 − x 4 + 2 x 2 − x Solución Factorizamos x en el polinomio y lo expresamos como P( x ) = x ( 2 x 4 − x 3 + 2 x − 1) Utilizamos la Regla de Ruffini para factorizar en el paréntesis

±1   ±1, ± 2   1 PRR = ± 1 , ±  2 

Posibles raíces o ceros: PRR = 

1 2

2

−1

2 2

−1

0

2

−1

1 0 −2 −2

0 0 2 2

0 2 −2 0

1 0

El polinomio factorizado es igual a

1 P( x ) = 2 x 5 − x 4 + 2 x 2 − x = x (2 x 4 − x 3 + 2 x − 1) = x ( x − )( x + 1)(2 x 2 − 2 x + 2) 2 También llegamos a factorizarlo de la siguiente manera:

1 2

Como ( x − ) es factor de P( x )

1 2 P( x ) = x (2 x − 1) (x + 1)( x 2 − x + 1)

Tenemos P( x ) = x ( x − ) (x + 1)( 2 x 2 − 2 x + 2) = x (

2x − 1 ) (x + 1) 2(x 2 − x + 1) 2

E. Método de los artificios: a) Cambio de variable o agrupaciones convenientes Mediante transformaciones u operaciones adecuadas se pueden lograr expresiones iguales para luego proceder a un cambio de variable, de tal manera que se obtenga una forma de factorización mas simple por los métodos ya estudiados. Ejemplos 1. Factorizar F = ( x − 2)( x − 1)( x + 2)( x + 3) + 3 Solución Agrupando adecuadamente y efectuando en la forma indicada se tiene

F = [ ( x − 2)( x + 3)

][ (x + 2)(x − 1) ] + 3

35


DAVID GONZÁLES LÓPEZ

F = ( x 2 + x − 6)( x 2 + x − 2) + 3 Haciendo: x 2 + x = m F(m) = (m − 6)(m − 2) + 3 F(m) = (m 2 − 8m + 15) = (m − 3)(m − 5) Reemplazando y escribiendo en términos de x F( x ) = ( x 2 + x − 3)( x 2 + x + 5) 2. Factorizar F = 1 + x ( x + 1)( x + 2)( x + 3) Solución Agrupando adecuadamente y efectuando en la forma indicada se tiene

F = 1 + [ x ( x + 3) ][ ( x + 1)( x + 2) ] F = 1 + ( x 2 + 3x )( x 2 + 3x + 2) Haciendo: x 2 + 3x = a F(a ) = 1 + a (a + 2) = a 2 + 2a + 1 = (a + 1) 2 Reemplazando y escribiendo en términos de x

F( x ) = ( x 2 + 3x + 1) 2 3. Factorizar F = ( x 2 + x )( x 2 + 5x + 6) + ( x 2 + 3x + 1) 2 + 1 Solución Expresando en su forma de factores al primer sumando

F = x ( x + 1)( x + 2)( x + 3) + ( x 2 + 3x + 1) 2 + 1 Agrupando y efectuando en la forma indicada

F = ( x 2 + 3x )( x 2 + 3x + 2) + ( x 2 + 3x + 1) 2 + 1 Haciendo:

x 2 + 3x = m F(m) = m(m + 2) + (m + 1) 2 + 1 = 2m 2 + 4m + 2 = 2(m 2 + 2m + 1) = 2(m + 1) 2

Reemplazando y escribiendo en términos de x

F( x ) = 2( x 2 + 3x + 1) 2 4. Factorizar F( x , y) = ( x + y + 1) 4 − 5( x + y) 2 − 10( x + y) − 1 Solución Haciendo:

x + y +1 = m ⇒ x + y = m −1 F(m) = m 4 − 5(m − 1) 2 − 10(m − 1) − 1

F(m) = m 4 − 5m 2 − 10m − 5 − 10m + 10 − 1 = m 4 − 5m 2 + 4 = (m 2 − 1)(m 2 − 4) F(m) = (m − 1)(m + 1)(m − 2)(m + 2) Reemplazando y escribiendo en términos de x e y F( x , y) = ( x + y + 1 − 1)( x + y + 1 + 1)( x + y + 1 − 2)( x + y + 1 + 2) F( x , y) = ( x + y)( x + y + 2)( x + y − 1)( x + y + 3)

36


DAVID GONZÁLES LÓPEZ

b) Quita y pon o reducción a diferencia de cuadrados Consiste en sumar y restar una misma expresión en forma conveniente de modo tal que al hacer agrupaciones, el objetivo, sea llegar a una diferencia de cuadrados. Ejemplos 1. Factorizar F( x ) = x 4 + 4 Solución 2 2 4 2 Sabemos que: ( x + 2) = x + 4 x + 4 Quitando y poniendo 4 x 2

F( x ) = x 4 + 4 + 4 x 2 − 4 x 2 F( x ) = ( x 4 + 4 x 2 + 4) − 4 x 2 El paréntesis es un trinomio cuadrado perfecto

F( x ) = ( x 2 + 2) 2 − 4 x 2 Por lo tanto la expresión factorizada es:

F( x ) = ( x 2 − 2 x + 2)( x 2 + 2 x + 2) 2. Factorizar F( x ) = 25x 4 + 11x 2 + 4 Solución Sabemos que: (5x + 2) = 25x + 20 x + 4 2

2

2

Quitando y poniendo 9 x 2

F( x ) = 25x 4 + 11x 2 + 4 + 9 x 2 − 9 x 2 F( x ) = (25x 4 + 20 x 2 + 4) − 9 x 2 El paréntesis es un trinomio cuadrado perfecto

F( x ) = (5x 2 + 2) 2 − 9 x 2 Por lo tanto la expresión factorizada es:

F( x ) = (5x 2 − 3x + 2)(5x 2 + 3x + 2) 3. Factorizar F( x , y) = 4 x 4 + 4 xy 2 − y 4 + 1 Sabemos que: (2 x + 1) = 4 x + 4 x + 1 2

2

4

2

Solución y (2 x − y 2 ) 2 = 4 x 2 − 4xy 2 + y 4

Quitando y poniendo 4 x 2

F( x , y) = 4 x 4 + 4 xy 2 − y 4 + 1 + 4 x 2 − 4 x 2 Agrupando en forma adecuada

F( x , y) = (4 x 4 + 4 x 2 + 1) + (−4 x 2 + 4 xy 2 − y 4 ) F( x , y) = (4 x 4 + 4 x 2 + 1) − (4 x 2 − 4 xy 2 + y 4 ) Factorizando ambos paréntesis ( trinomio cuadrado perfecto)

F( x ) = (2 x 2 + 1) 2 − (2 x − y 2 ) 2 Por lo tanto la expresión factorizada es:

F( x ) = (2 x 2 − 2 x + 1 + y 2 ) (2 x 2 + 2 x + 1 − y 2 ) c) Sumas y restas especiales Consiste en sumar y restar una expresión en forma conveniente de modo tal que se obtenga por lo general ( x 2 + x + 1) o ( x 2 − x + 1) ambos componentes de una diferencia o suma de

37


DAVID GONZÁLES LÓPEZ

cubos; en otros casos se puede buscar otro tipo de expresiones que conduzcan a la factorización del polinomio. Ejemplos 1. Factorizar F( x ) = x 5 + x + 1 Solución 2 Sumamos y restamos x

F( x ) = x 5 + x + 1 + x 2 − x 2 Agrupamos en forma adecuada y factorizamos

F( x ) = ( x 5 − x 2 ) + ( x 2 + x + 1) F( x ) = x 2 ( x 3 − 1) + ( x 2 + x + 1) F( x ) = x 2 ( x − 1)( x 2 + x + 1) + ( x 2 + x + 1) Extrayendo factor común ( x 2 + x + 1) la expresión queda factorizada

[

F( x ) = ( x 2 + x + 1) x 2 ( x − 1) + 1

]

F( x ) = ( x + x + 1)( x − x + 1) 2

3

2

2. Factorizar F( x ) = a 5 + a − 1 Solución Primera forma Sumamos y restamos a 2

F( x ) = a 5 + a − 1 + a 2 − a 2 Agrupamos en forma adecuada y factorizamos

F( x ) = (a 5 + a 2 ) + (−a 2 + a − 1) F( x ) = a 2 (a 3 + 1) − (a 2 − a + 1) F( x ) = a 2 (a + 1)(a 2 − a + 1) − (a 2 − a + 1) Extrayendo factor común (a 2 − a + 1) la expresión queda factorizada

[

F( x ) = (a 2 − a + 1) a 2 (a − 1) − 1

]

F( x ) = (a − a + 1)(a + a − 1) 2

3

2

Segunda forma Completando el polinomio sumando y restando: a 4 + a 3 + a 2

F( x ) = a 5 + a − 1 + a 4 + a 3 + a 2 − a 4 − a 3 − a 2 Ordenando convenientemente y factorizando tenemos

F( x ) = (a 5 − a 4 + a 3 ) + (a 4 + a 2 − a 3 ) + (a − 1 − a 2 ) F( x ) = a 3 (a 2 − a + 1) + a 2 (a 2 − a + 1) − (a 2 − a + 1) F( x ) = (a 2 − a + 1) (a 3 + a 2 − 1) 3. Factorizar F( x ) = x 10 + x 8 + 1 Solución Primera forma Haciendo x 2 = m

⇒ F(m) = m 5 + m 4 + 1 Sumando y restando m 2 y luego agrupando 38


DAVID GONZÁLES LÓPEZ

F(m) = m 5 + m 4 + 1 + m 2 − m 2 F(m) = m 2 (m 3 − 1) + (m 4 + m 2 + 1) Factorizamos m 3 − 1 y m 4 + m 2 + 1 ( Identidad de ARGAND) F(m) = m 2 (m − 1)(m 2 + m + 1) + (m 2 + m + 1)(m 2 − m + 1) Luego, la expresión factorizada es

F(m) = (m 2 + m + 1)(m 3 − m + 1) Reemplazando el valor de m y escribiendo en términos de x F( x ) = ( x 4 + x 2 + 1)( x 6 − x 2 + 1) F( x ) = ( x 2 + x + 1)( x 2 − x + 1)( x 6 − x 2 + 1) Segunda forma Completando con: x 6 , x 4 , x 2

F( x ) = x 10 + x 8 + 1 + x 6 + x 4 + x 2 − x 6 − x 4 − x 2 F( x ) = x 10 + x 8 + x 6 + x 4 + x 2 + 1 − x 6 − x 4 − x 2 F( x ) = x 6 ( x 4 + x 2 + 1) + ( x 4 + x 2 + 1) − x 2 ( x 4 + x 2 + 1) F( x ) = ( x 4 + x 2 + 1)( x 6 + 1 − x 2 ) = ( x 2 + x + 1)( x 2 − x + 1)( x 6 − x 2 + 1) Ejercicios 06: Factorización Factor común y/o agrupación de términos 1. F = x 7 − 2 x 6 + x 4 − 2 x 3 2. F = x ( x 2 − y 2 + xz) − y 2 z 3. F = x 3 − 2 x 2 y + xy 2 − 2 y 3 4. F = x 3 − xy 2 + x 2 y − y 3 + x 2 − y 2 5. F = (z − x − y)(2a − b) − ( x + y − z)(a + 2b) 6. F = x n + p + x n y p + y m x p + y m + p 7. F = 3( m − n ) 2 ( m + n ) − 3( m + n ) 2 ( m − n ) − m( m + n )(m − n ) 8. F = p(q 2 + r 2 ) + q ( r 2 + p 2 ) 9. F = mn ( x 2 − y 2 ) + xy( m 2 − n 2 ) 10. F = ( m + 2)(m + 3)(m + 4) + ( m + 3)(m + 4) + ( m + 4) 11. F = x y y x + xy + x y +1 + y x +1 12. F = x m + a + x m y b + x a y n + y n + b + z p x a + z p y b Identidades 13. F = (3x + y) 2 − (3y − x ) 2 14. F = (ax − 3b) 2 − ( bx − 3a ) 2 15. F = m 2 − 4p 2 + 4mn + 4n 2 16. F = x 7 + a 3 x 4 − a 4 x 3 − a 7 17. F = 1 + 3a 2 − 3a − a 3 18. F = x 3 y 2 + y 3 z 2 − x 3 z 2 − y 5

39


DAVID GONZÁLES LÓPEZ

19. F = m 2 + 2m + mn + n + 1 20. F = (1 + mx ) 2 − ( m + x ) 2 21. F = 8 − 12a 2 − 6a 4 − a 6 22 F = x 2 + 4 xy + 4 y + 2 x + 4 y 2 Aspa simple 23. F = 8x 2 + 10 x − 3 24. F = 15x 4 + 2 x 2 − 8 25. F = 5x 6 − 14 x 3 − 3 26. F = 4a 4 b − 4a 3 b 2 − 24a 2 b 3 27. F = 12( x − y) 2 + 13( x − y) − 4 28. F = 6 x 2 + 19 xy + 15 y 2 29. F = x 2 a + 4 + 5x a + 4 − 50 x 4 30. F = ( x − 1) 4 + ( x − 1) 2 − 6 31. F = 4 x 2 m + 2 − 4 x m + 2 − 3x 2 32. F = 3 2 x + 2 − 3 x +1 − 30 Regla de Ruffini 33. F = x 3 − 3x 2 − x + 3 34. F = x 3 + 2 x 2 − x − 2 35. F = x 3 + 6 x 2 + 11x + 6 36. F = x 4 + 3x 3 − 3x 2 + 3x − 4 37. F = x 5 + 4 x 4 − 10 x 2 − x + 6 38. F = x 3 + 6 x 2 + 15x + 14 39. F = 2 x 3 − 5x 2 − 4 x + 3 40. F = 12 x 3 + 16 x 2 + 7 x + 1 41. F = 2 x 5 − x 4 − 10 x 3 + 5x 2 + 8x − 4 42. F = 12 x 5 − 8x 4 − 13x 3 + 9 x 2 + x − 1 43. F = 8x 3 − 12 x 2 + 6 x − 65 44. F = 30 x 3 + 19 x 2 − 1 Cambio de variable o agrupaciones convenientes 45. F = ( x − 3)( x − 2)( x + 1)( x + 2) − 12 46. F = ( x + 1)( x + 2)( x + 3)( x + 4) + 1 47. F = ( x − 2)( x + 3)( x + 2)( x − 1) + 3 48. F = ( x + 1) 2 ( x − 3)( x + 5) + 63 49. F = −10 + x ( x − 3)( x − 2)( x + 1) 50. F = ( x + 1)( x − 2)( x + 3)( x − 4) + 24 51. F = ( x 2 + x + 1) 2 + 3x 2 + 3x − 15 52. F = ( x − 1)( x + 2)( x − 3)( x − 6) + 7 x 2 − 28x + 1 53. F = ( x − 2) 2 ( x 2 − 4 x + 6) − 15

40


DAVID GONZÁLES LÓPEZ

54. F = ( x 2 + x )( x 2 + 5x + 6) + ( x 2 + 3x + 1) 2 + 1 Quita y pon o reducción a diferencia de cuadrados 55. F = x 4 + x 2 + 1 56. F = m 4 + m 2 n 2 + n 4 57. F = m 4 + 2m 2 n 2 + 9m 4 58. F = x 8 − 12 x 4 + 16 59. F = 4 x 4 + 3x 2 y 2 + y 4 60. F = x 4 + 2 x 2 + 9 61. F = x 4 + 4 y 4 62. F = a 4 b 4 + 64c 4 63. F = x 4 + x 2 y 2 + 49 y 4 64. F = x 4 + y 4 − 27 x 2 y 2 Sumas y restas especiales 65. F = a 5 + a + 1 66. F = x 5 + x − 1 67. F = x 7 + x 5 − 1 68. F = (a + 1) 5 + a + 2 69. F = ( m + 1) 5 + m 70. 71. 72. 73. 74.

F = x5 + x4 +1 F = x 10 + x 2 + 1 F = m10 + m 8 + 1 F = x8 − x +1 F = x 10 + x 2 − 1

Factorizar y simplificar 75. E =

( x + y) 4 − ( x − y ) 4 8x 3 y + 8xy 3

76. E =

( x + y)( x 3 − y 3 ) + ( x − y)( x 3 + y 3 ) x 4 − y4

77. E = (

78. E =

x 2 − 64 x 2 − 12 x − 64 )( ) x 2 − 24 x + 128 x 2 − 4 x − 32

x 2 − xy − y − 1 x 2 − xy − 2 x + y + 1

a 2 + 2ab + b 2 − c 2 a 2 − 2ac + c 2 − b 2 x a 2 − b 2 − c 2 − 2bc b 2 − 2bc + c 2 − a 2  a(a 3 − b 3 )   b(a 3 − b 3 )  80. E =  +  3 3 3  3   a −b   a −b 

79. E =

41


DAVID GONZÁLES LÓPEZ

81. E =

x6 + x3 − 2 x 4 − x3y − x + y

82. E =

(3x − 1) 3 − 27 x + 9 (3x + 2)(3x − 4)

83. E =

(3x 3 − 3x )( x 3 − 1) ( x 4 + x 3 + x 2 )( x 2 − 1)

(1 + xy) 2 − ( x + y) 2 1− x2 (1 + x 2 ) 4 + (1 − x 4 ) 2 85. E = (1 + x 2 + x 4 ) 2 − x 4 84. E =

86. E =

x ( y 2 + z 2 − x 2 ) + y(z 2 + x 2 − y 2 ) z 2 − x ( x − 2 y) − y 2

87. E =

( m + n ) 2 ( x + y ) 2 − ( m − n ) 2 ( x − y) 2 mn ( x 2 + y 2 ) + xy(m 2 + n 2 )

a 4 − 3a 2 b 2 + b 4 a 2 − ab − b 2 2 ( m + n ) 2 + ( m − n ) 2 − 4( m 2 − n 2 ) 2 89. E = (m 2 + n 2 ) 2 − (n 2 − m 2 ) 2

88. E =

[

]

 a

b

a

b  

90. E =  ( + ) 2 + ( − ) 2  b a b a



2

2  a b  − 4  ( )2 - ( )2  a   b

1.5. FRACCIONES ALGEBRAICAS RACIONALES Fracción algebraica es el cociente indicado de dos expresiones algebraicas racionales en donde al menos una letra o variable figura en el denominador. Notación:

P , donde P y Q son polinomios enteros Q

Ejemplos

x + y 3 x + y 2 x 2 + 3y 4 a −1 , , , , x−y x z x−z a+b+c Clases de fracciones Existen las siguientes clases de fracciones algebraicas: A. Fracción propia Se caracteriza porque el grado del numerador es menor que el grado del denominador Ejemplos:

x3 − 3 x3 − 3 , x4 +1 x4 +1

y

xy 3 + 2 ( respecto de x ) x3 + y2

B. Fracción impropia Se caracteriza porque el grado del numerador es mayor o igual que el grado del denominador .

42


DAVID GONZÁLES LÓPEZ

x5 + x3 x2 + 2

,

x 3 − 2x + 1 4x 3 + 5

y

x + y4 ( respecto de y ) x 2 y2 − 3

Observación Toda fracción algebraica racional impropia se puede convertir en la suma de un polinomio (cociente) con una fracción la cual es propia a través de la división. Así: Sea

P es una fracción racional impropia Q

P R Ejemplo

Q C

P = Q.C + R

P R =C+ Q Q

, donde

R es fracción propia Q

3x 3 + 2 x 2 − 3 3x − 1 = (3x + 2) + 2 2 x −1 x −1

C. Fracciónes Homogeneas Son fracciones que tienen el mismo denominador

x2 x +3 4x 2 x 3 − 5x + 1 , 2 , , ( x − 1)( x + 1) x 2 −1 x −1 − (1 − x 2 ) D. Fracción Heterogeneas Son fracciones que tiene diferente denominador

2x + 3 5x 2x 3 − 5 x2 + 2 , 2 , , 2 x −1 3x + 2 x +1 x +4 E. Fracciones equivalentes Son aquellas que teniendo formas diferentes se caracterizan porque siempre tendrán los mismos valores numéricos, para cualquier valor asignado a sus variables, a excepción de aquellos que hagan cero el denominador

x +3 1 ≡ ; ∀x ≠ −3,−4 x + 7 x + 12 x + 4 3 1 Así por ejemplo para x = 0 ⇒ = 12 4 4 1 para x = 1 ⇒ = 20 5 Ejemplo

2

F. Fracciones complejas o compuestas Se caracterizan porque en su numerador o denominador, o en ambos, aparecen otras fracciones algebraicas.

x +1 Ejemplos: x + 2 , 3x

5x , 3x − 1 x+2

x x+2 , x −1 2x + 5

x−2 4 , x 3

x y − y x x +1 , y 7 1+ x+ x 1+ x2 x− 2 x −1

43


DAVID GONZÁLES LÓPEZ

G. Fracciones continuas Es un caso particular de las fracciones complejas, que se caracterizan porque en el numerador de cada fracción siempre esta la unidad.

1

Ejemplo

1

x− x−

1 x +1

H. Fracción irreductible Son aquellas fracciones que se caracterizan porque en el numerador y denominador aparecen expresiones que no tienen ningún factor común( el numerador y denominador son primos entre sí) es decir no admiten simplificación. Ejemplos

x+3 , x +1

5 x

,

3x 2

Signos de una fracción Toda fracción posee tres signos : signo del numerador, signo del denominador y signo de la fracción. El cambio de dos signos de una fracción no altera el signo total de la fracción Ejemplos

P −P = Q −Q P −P P F=− = = Q Q −Q +P −P +P −P F=+ =− =− =+ −Q +Q +Q −Q −P +P −P +P G=− =− =+ =+ −Q −Q +Q +Q a−b (b − a ) a−b H= =− =− = −1 b−a b−a (a − b ) (a − b)(a − c) − (b − a )(a − c) (b − a )(a − c) J= = = =1 (b − a )(c − a ) − (b − a )(a − c) (b − a )(a − c) 5 5 5 5 K= + = − =0 x −3 3− x x −3 x −3

1. F = 2. 3. 4. 5. 6. 7.

Observaciones - En toda fracción podrán efectuarse tres formas de jugar con los signos y tendremos como resultado otras fracciones equivalentes. Ejemplo

2−x x−2 2−x x−2 =+ =+ =− 7−x 7−x x −7 x −7 (1) (2) (3)

- Si a los componentes de una fracción se les multiplica o divide por una expresión diferente de cero tendremos como resultado otras fracciones equivalentes.

44


DAVID GONZÁLES LÓPEZ

Ejemplo

a a am m F= = = b bm b m

, ∀m ≠ 0

Simplificación de fracciones Simplificar una fracción es transformarla en otra equivalente e irreductible. Para simplificar una fracción se sugiere descomponer, tanto numerador y denominador en sus factores primos(factorizarlos) para luego eliminar sus factores comunes. Ejemplo Simplificar:

x 4 + x 2 + 1 ( x 2 + x + 1)( x 2 − x + 1) x 2 + x + 1 F= = = x +1 x3 +1 ( x + 1)( x 2 − x + 1)

Operaciones con fracciones - Para sumar o restar fracciones es necesario hallar el mínimo común múltiplo de los denominadores. Ejemplos 1. Efectuar y simplificar: M =

2 3 4x − 7 + − 2 x −3 x +2 x −x −6 Solución

2 3 4x − 7 2 3 4x − 7 M= + − 2 = + − x − 3 x + 2 x − x − 6 x − 3 x + 2 ( x − 3)( x + 2) M=

el m.c.m. es ( x − 3)( x + 2)

2( x + 2) + 3( x − 3) − (4 x − 7) 2 x + 4 + 3x − 9 − 4 x + 7 x+2 1 = = = ( x − 3)( x + 2) ( x − 3)( x + 2) ( x − 3)( x + 2) x − 3

2. Efectuar y simplificar: M =

1 4x 3x 2 − 2 + 3 x −1 x −1 x −1 Solución

2

1 4x 3x 1 4x 3x 2 M= − + = − + x − 1 x 2 − 1 x 3 − 1 x − 1 ( x − 1)( x + 1) ( x − 1)( x 2 + x + 1) el m.c.m. es ( x + 1)( x − 1)( x 2 + x + 1)

x 3 + x 2 + x + x 2 + x + 1 − 4 x 3 − 4 x 2 − 4 x + 3x 3 + 3x 2 x 2 − 2x + 1 = ( x − 1)( x + 1)( x 2 + x + 1) ( x − 1)( x + 1)( x 2 + x + 1) ( x − 1)( x − 1) x −1 M= = 2 ( x − 1)( x + 1)( x + x + 1) ( x + 1)( x 2 + x + 1) M=

3. Efectuar y simplificar: M =

3 1 1 − − 2x + 2 4 x − 4 2 − 2x 2

45


DAVID GONZÁLES LÓPEZ

Solución

3 1 1 M= − + 2 2x + 2 4 x − 4 2x − 2 3 1 1 3 1 1 M= − + = − + 2 2( x + 1) 4( x − 1) 2( x − 1) 2( x + 1) 4( x − 1) 2( x − 1)( x + 1) el m.c.m. es 4( x − 1)( x + 1) M=

6( x − 1) − ( x + 1) + 2 6 x − 6 − x − 1 + 2 5x − 5 5( x − 1) 5 = = = = 4( x + 1)( x − 1) 4( x − 1)( x + 1) 4( x − 1)( x + 1) 4( x − 1)( x + 1) 4( x + 1)

4. Efectuar y simplificar: M =

4 5 1 − + 2 2 12 − 12 x + 3x 4( 4 + 4 x + x ) 8 − 2 x 2 Solución

4 5 1 − − 2 2 3( x − 4 x + 4) 4( x + 4 x + 4) 2( x − 4) 4 5 1 M= − − 2 2 2( x − 2)( x + 2) 3( x − 2) 4( x + 2) M=

2

el m.c.m. es 12( x − 2) 2 ( x + 2) 2

M=

16( x + 2) 2 − 15( x − 2) 2 − 6( x 2 − 4) 16( x 2 + 4 x + 4) − 15( x 2 − 4 x + 4) − 6 x 2 + 24 = 12( x − 2) 2 ( x + 2) 2 12( x − 2) 2 ( x + 2) 2

M=

16 x 2 + 64 x + 64 − 15x 2 + 60 x − 60 − 6 x 2 + 24 − 5x 2 + 124 x + 28 = 12( x − 2) 2 ( x + 2) 2 12( x − 2) 2 ( x + 2) 2

- Para multiplicar fracciones se multiplican numeradores y denominadores entre si. Ejemplos 1. Simplificar M = (

5x + 25 7 x + 7 )( ) 14 10 x + 50 Solución

M=(

5x + 25 7 x + 7 (5x + 25)(7 x + 7) 5( x + 5) 7( x + 1) x + 1 )( )= = = 14 10 x + 50 14(10 x + 50) 14(10)( x + 5) 4

2. Simplificar M = (

x 2 − 5x + 6 6x x 2 − 25 )( 2 )( ) 3x − 15 x − x − 30 2 x − 4 Solución

( x − 5x + 6)(6 x )( x − 25) ( x − 3)( x − 2)(6 x )( x − 5)( x + 5) x ( x − 3) = = 2 x −6 (3x − 15)( x − x − 30)(2 x − 4) 3( x − 5)( x − 6)( x + 5) (2 ( x − 2)) 2

M=

2

3. Simplificar M = ( x + 3 −

5 5 )( x − 2 + ) x −1 x+4 46


DAVID GONZÁLES LÓPEZ

Solución

5 5 x + 2x − 8 x 2 + 2x − 3 ( x 2 + 2 x − 8)( x 2 + 2 x − 3) M = (x + 3 − )( x − 2 + )=( )( )= x −1 x+4 x −1 x+4 ( x − 1)( x + 4) ( x + 4)( x − 2)( x + 3)( x − 1) M= = ( x − 2)( x + 3) ( x − 1)( x + 4) 2

- Para dividir fracciones, se invierte la fracción del divisor y se procede como en la multiplicación. Ejemplos 1. Simplificar M =

x3 − x 5x 2 − 5 x ÷ 2x + 6 2x 2 + 6x Solución

x3 − x 5 x 2 − 5x x3 − x 2x + 6 ( x 3 - x)(2x + 6) x ( x 2 − 1) 2( x + 3) ÷ = ( )( ) = = 2x + 6 2x 2 + 6x 2 x 2 + 6 x 5x 2 − 5x (2 x 2 + 6 x )(5x 2 − 5x ) 2 x ( x + 3) 5x ( x − 1) x ( x − 1)( x + 1) 2( x + 3) ( x + 1) = M= 2 x ( x + 3) 5x ( x − 1) 5x

M=

2. Simplificar M =

x 3 + 125 x 3 − 5x 2 + 25x ÷ x 2 − 64 x 2 + x − 56 Solución

M=

x 3 + 125 x 3 − 5x 2 + 25x x 3 + 125 x 2 + x − 56 ( x 3 + 125)( x 2 + x − 56) ÷ = ( ) ( ) = x 2 − 64 x 2 + x − 56 x 2 − 64 x 3 − 5x 2 + 25x ( x 2 − 64)( x 3 − 5x 2 + 25x )

M=

( x + 5)( x 2 − 5x + 25)( x + 8)( x − 7) ( x + 5)( x + 7) = x ( x − 8) ( x − 8)( x + 8) x ( x 2 − 5x + 25)

3. Simplificar M = ( x −

2x − 1 x −1 ) ÷ (x 2 + 1 − ) 2 x x +2 Solución

2x − 1 x −1 x 3 + 2x − 2x + 1 x3 + x − x +1 x3 +1 x3 +1 2 ) ) ÷ ( x + 1 − ) = ( ) ÷ ( ) = ( ) ÷ ( x x x x2 + 2 x2 + 2 x2 + 2 x3 +1 x ( x 3 + 1) x x M=( 2 )( 3 ) = 2 = 2 3 x + 2 x + 1 ( x + 2)( x + 1) x + 2 M = (x −

Simplificación de fracciones complejas Regla - Se efectúan las operaciones indicadas en el numerador y denominador de la fracción compleja - Se divide el resultado que se obtenga en el numerador entre el resultado que se obtenga en el denominador. Ejemplos

47


DAVID GONZÁLES LÓPEZ

3 x − x 3 1. simplificar M = 3 1+ x Solución

3 x 9 − x2 − = x 3 3x 3 x+3 Efectuando en el denominador: 1 + = x x

Efectuando en el numerador:

Dividiendo tenemos

9 − x2 2 3x = (9 − x ) x = (3 − x )(3 + x ) = 3 − x x +3 3x ( x + 3) 3x ( x + 3) 3x x 12 x −1− x−2 2. simplificar M = 16 x +6+ x−2 Solución Efectuando en el numerador y denominador tenemos:

12 ( x − 1)( x − 2) − 12 x 2 − 3x − 10 ( x 2 − 3x − 10)( x − 2) ( x 2 − 3x − 10) x − 2 x − 2 x − 2 M= = = 2 = 2 = 2 16 ( x + 6)( x − 2) + 16 x + 4x + 4 ( x + 4 x + 4)( x − 2) ( x + 4 x + 4) x+6+ x−2 x−2 x−2 ( x − 5)( x + 2) x − 5 M= = ( x + 2)( x + 2) x + 2 x −1−

1

3. simplificar M =

1+

1 1−

1 x

Solución Este tipo de fracciones se efectúa de abajo hacia arriba, como veremos en el desarrollo

1

M= 1+

1 1−

1

= 1 x

1+

1 x −1 x

1

= 1+

x x −1

=

1 1 x −1 = = x − 1 + x 2x − 1 2x − 1 x −1 x −1

Fracciones parciales Hemos estudiado que dada una suma o resta de fracciones algebraicas racionales se pueden reducir a una sola fracción, cuyo denominador es el mínimo común múltiplo de las fracciones dadas.

48


DAVID GONZÁLES LÓPEZ

Así tenemos:

5 1 5( x + 2) + ( x − 3) 6x + 7 + = = x −3 x +2 ( x − 3)( x + 2) ( x − 3)( x + 2) 3 x 3( x − 1) − x 2x − 3 (2) − = = 2 2 x − 1 ( x − 1) ( x − 1) ( x − 1) 2 (1)

(3)

2 1 2 2 x ( x 2 − x + 1) + x ( x + 1) − 2( x + 1)( x 2 − x + 1) − x 2 + 3x − 2 + 2 − = = x +1 x − x +1 x x ( x + 1)( x 2 − x + 1) x ( x + 1)( x 2 − x + 1)

En algunos casos es necesario realizar el proceso inverso, es decir dada una fracción algebraica racional escribirla como una suma de fracciones parciales o simples, a este proceso se le llama descomposición de una fracción en suma de fracciones parciales. Para descomponer una fracción en fracciones parciales, es necesario tener en cuenta las siguientes consideraciones: (1) La fracción dada debe ser una fracción propia, es decir, el grado del numerador debe ser menor que el grado del denominador, si no lo fuera, se efectúa la división, de modo que se obtenga un polinomio entero más una fracción propia. Así:

x 3 + 2x 2 − 1 4x + 7 = x+2+ 2 2 x −4 x −4

(2) La fracción debe ser una fracción irreductible, si no lo fuera, se realiza la simplificación. Así:

x2 + x − 2 ( x − 1)( x + 2) x+2 = = 2 2 2 ( x − 1)( x + x − 6) ( x − 1)( x + x + 6) x + x − 6

(3) La fracción dada debe tener como denominador un polinomio que pueda ser factorizado. Por lo general el número de factores del denominador da el número de fracciones simples o parciales en que se descompone la fracción dada. Así:

4 x 2 + 3x − 5 4 x 2 + 3x − 5 = x 3 + 3x 2 − x − 3 ( x + 1)( x − 1)( x + 3)

debe tener tres fracciones parciales

En la descomposición de fracciones parciales se presentan los siguientes casos: Primer caso: Denominador con factores de primer grado no repetidos. Para cada factor de primer grado diferente de la forma ( x ± a ) en el denominador, corresponde una fracción parcial de la forma:

A x±a

,

donde A es una constante a determinar.

Segundo caso: Denominador con factores de primer grado repetidos. - Para cualquier factor lineal ( x ± a ) que se presenta dos veces ( x ± a ) 2 en el denominador, corresponde dos fracciones parciales de la forma:

[

]

A2 A1 y , donde A 1 y A 2 son las constantes a determinar. x±a (x ± a ) 2

[

]

- Si ( x ± a ) se presenta tres veces ( x ± a ) 3 en el denominador, corresponde tres fracciones parciales de la forma:

49


DAVID GONZÁLES LÓPEZ

A3 A2 A1 , y , donde A 1 , A 2 y A 3 son las constantes a determinar 2 x ± a (x ± a ) (x ± a ) 3

[

- Si ( x ± a ) se presenta n veces ( x ± a ) n parciales de la forma:

] en

el denominador, corresponde n fracciones

A3 A2 An A1 , , ,…, , donde A 1 , A 2 , A 3 , … , A n son las constantes 2 3 x ± a (x ± a ) (x ± a ) (x ± a ) n a determinar Tercer caso: Denominador con factores de segundo grado no repetidos. Para cada factor cuadrático diferente de la forma ( x 2 + ax + b) en el denominador, corresponde una fracción parcial de la forma:

Ax + B x + ax + b 2

,

donde A y B son constantes a determinar.

Cuarto caso: Denominador con factores de segundo grado repetidos. - Para cualquier factor cuadrático ( x 2 + ax + b) que se presenta dos veces ( x 2 + ax + b) 2 en el denominador, corresponde dos fracciones parciales de la forma:

[

A 2 x + B2 A1 x + B1 y 2 x + ax + b ( x 2 + ax + b) 2

]

, donde A 1 , A 2 , B1 y B 2 son las constantes a determinar.

[

]

- Si ( x 2 + ax + b) se presenta tres veces ( x 2 + ax + b) 3 en el denominador, corresponde tres fracciones parciales de la forma:

A 3 x + B3 A 2 x + B2 A1 x + B1 , y 2 2 2 x + ax + b ( x + ax + b) ( x 2 + ax + b) 3

, donde A 1 , A 2 , A 3 , B1 , B 2 y B 3 son las

constantes a determinar. - Si ( x 2 + ax + b) se presenta n veces ( x 2 + ax + b) n fracciones parciales de la forma:

[

]

en el denominador, corresponde n

A 3 x + B3 A 2 x + B2 A n x + Bn A1 x + B1 , , ,…, 2 2 2 2 3 x + ax + b ( x + ax + b) ( x + ax + b) ( x 2 + ax + b) n donde A 1 , A 2 , A 3 , …, A n , B1 , B 2 , B 3 , …, B n son las constantes a determinar, cuyos valores se calculan utilizando las condiciones de los polinomios idénticos ( dando valores adecuados a la variable) Observación - Los factores lineales que aparecen en el denominador de las fracciones a descomponer pueden ser de la forma (ax ± b) con a ≠ 1 . - Los factores cuadráticos que aparecen en el denominador de las fracciones a descomponer pueden ser de la forma (ax 2 + bx + c) con a ≠ 1 . Ejemplos 1. Descomponer en fracciones parciales:

6x + 7 x −x−6 2

50


DAVID GONZÁLES LÓPEZ

Solución Factorizando el denominador

6x + 7 6x + 7 = x − x − 6 ( x − 3)( x + 2) 2

Como se tiene en el denominador dos factores lineales no repetidos, a cada uno le corresponde una fracción simple, entonces:

6x + 7 A B = + x −x −6 x −3 x +2 donde A y B son independientes de x cuyos valores debemos hallar. 2

Efectuando en el segundo miembro

6x + 7 A( x + 2) + B( x − 3) = x −x−6 x2 − x − 6 2

Quitando denominador tenemos

6 x + 7 = A( x + 2) + B( x − 3)

Aplicando criterio de polinomios idénticos y dando valores convenientes a x para hallar las constantes A y B tenemos: Para x = −2 se tiene 6(−2) + 7 = −5B ⇒ B = 1 Para x = 3 se tiene 6(3) + 7 = 5A ⇒ A=5 Luego

6x + 7 5 1 = + x −x −6 x −3 x +2 2

2. Descomponer en fracciones parciales:

5x − 6 2 x + 5x − 3 2

Solución Factorizando el denominador

5x − 6 5x − 6 = 2 x + 5x − 3 (2 x − 1)( x + 3) 2

Como se tiene en el denominador dos factores lineales no repetidos, a cada uno le corresponde una fracción simple, entonces:

5x − 6 A B = + 2 x + 5x − 3 2 x − 1 x + 3 2

Efectuando en el segundo miembro

5x − 6 A( x + 3) + B(2 x − 1) = 2x − 1 2 x + 5x − 3 2

Quitando denominadores tenemos

5x − 6 = A( x + 3) + B(2 x − 1) Damos valores a x para determinar A y B Para x = −3 se tiene 5( −3) − 6 = −7 B ⇒ B = 3 1 1 7 Para x = se tiene 5( ) − 6 = A ⇒ A = −1 2 2 2 5x − 6 −1 3 Luego = + 2 x 2 + 5x − 3 2 x − 1 x + 3 3. Descomponer

x 3 + 2x 2 − 1 en fracciones parciales x2 − 4 51


DAVID GONZÁLES LÓPEZ

Solución Por ser la fracción impropia, efectuamos la división y tenemos

x 3 + 2x 2 − 1 4x + 7 = x+2+ 2 2 x −4 x −4 Ahora descomponemos en fracciones parciales la fracción propia

4x + 7 x2 − 4

Factorizando el denominador

4x + 7 4x + 7 = 2 x − 4 ( x − 2)( x + 2) El denominador tiene dos factores lineales no repetidos, luego se tendrán dos fracciones parciales

4x + 7 A B = + ( x − 2)( x + 2) x − 2 x + 2 Efectuando en el segundo miembro

4x + 7 A( x + 2) + B( x − 2) = ( x − 2)( x + 2) ( x − 2)( x + 2) Quitando denominadores tenemos

4 x + 7 = A( x + 2) + B( x − 2) Damos valores a x para determinar A y B 1 4 15 Para x = 2 se tiene 4( 2) + 7 = 4A ⇒ A= 4 4x + 7 15 / 4 1 / 4 15 1 = + = + Luego ( x − 2)( x + 2) x − 2 x + 2 4( x − 2) 4( x + 2)

Para x = −2

Finalmente

se tiene 4( −2) + 7 = −4B ⇒

B=

x 3 + 2x 2 − 1 4x + 7 15 1 = x+2+ 2 = x+2+ + 2 4( x − 2) 4( x − 2) x −4 x −4

4. Descomponer en fracciones parciales:

4 x 2 − 15x + 8 x 3 − 3x 2 + 4 Solución

Factorizando el denominador

4 x 2 − 15x + 8 4 x 2 − 15x + 8 = x 3 − 3x 2 + 4 ( x + 1)( x − 2) 2 El denominador presenta un factor de primer grado no repetido y un factor de primer grado repetido dos veces, por lo cual corresponden tres fracciones parciales.

4 x 2 − 15x + 8 A B C = + + 3 2 x − 3x + 4 x + 1 ( x − 2) ( x − 2) 2 Desarrollando en el segundo miembro

4 x 2 − 15x + 8 A( x − 2) 2 + B( x + 1)( x − 2) + C( x + 1) = x 3 − 3x 2 + 4 ( x + 1)( x − 2) 2 52


DAVID GONZÁLES LÓPEZ

Aplicando criterio de polinomios idénticos y dando valores convenientes a x , hallamos las constantes A , B y C . Para x = 2 se tiene 4( 2) 2 − 15( 2) + 8 = 3C ⇒ C = −2 Para x = −1 Para x = 2

Luego

se tiene 4( −1) 2 − 15( −1) + 8 = 9A

⇒ A=3

se tiene 4(0) − 15(0) + 8 = 4A − 2B + C ⇒ 2

B =1

4 x 2 − 15x + 8 3 1 2 = + − 3 2 x − 3x + 4 x + 1 ( x − 2) ( x − 2) 2 x2 + 5 3x ( x − 1) 2

5. Descomponer en fracciones parciales:

Solución El denominador presenta un factor de primer grado no repetido y otro repetido, entonces se establece:

 x2 + 5 1 A B C =  + + 2 2  3  x x − 1 ( x − 1)  3x ( x − 1) También se puede escribir

x2 + 5 A( x − 1) 2 + Bx ( x − 1) + Cx = x ( x − 1) 2 x ( x − 1) 2 Eliminando denominador tenemos

x 2 + 5 = A( x − 1) 2 + Bx ( x − 1) + Cx Aplicando criterio de polinomios idénticos y dando valores convenientes a x para hallar las constantes A , B y C tenemos: Para x = 0 se tiene 5 = A Para x = 1 se tiene 1 + 5 = C ⇒ C=6 ⇒ B = −4 Para x = 2 se tiene 4 + 5 = A + 2B + 2C

Luego

 5 x2 + 5 1 5 −4 6 4 2 =  + + = − + 2 2  3  x x − 1 ( x − 1)  3x 3( x − 1) ( x − 1) 2 3x ( x − 1)

6. Transformar en fracciones parciales:

x 2 − 4x − 1 ( x − 1)( x 2 − 3)

Solución Por tener el denominador un factor de primer grado y uno de segundo grado, se originan las siguientes fracciones parciales

x 2 − 4x − 1 A Bx + C = + 2 2 ( x − 1)( x − 3) x − 1 x − 3 Efectuando en el segundo miembro

x 2 − 4x − 1 A( x 2 − 3) + (Bx + C)( x − 1) = ( x − 1)( x 2 − 3) ( x − 1)( x 2 − 3) Eliminando denominadores tenemos

53


DAVID GONZÁLES LÓPEZ

x 2 − 4 x − 1 = A( x 2 − 3) + (Bx + C)( x − 1) Hallamos los valores de A , B y C de dos formas Primera forma Para x = 1 se tiene − 4 = −2A ⇒ A = 2 Para x = 0 se tiene − 1 = −3A − C ⇒ C = −5 Para x = 2 se tiene − 5 = A + 2B + C ⇒ B = −1 Segunda forma

x 2 − 4 x − 1 = A( x 2 − 3) + (Bx + C)( x − 1) x 2 − 4 x − 1 = (A + B) x 2 + (−B + C) x − 3A − C Por identidad de polinomios igualamos los coeficientes de los términos de igual grado A + B =1 …… ( I ) − B + C = −4 …… ( II ) − 3A − C = −1 ….. ( III ) Resolviendo se obtiene A = 2 , B = −1 y C = −5

x 2 − 4x − 1 2 −x −5 2 x+5 = + 2 = − 2 Luego 2 ( x − 1)( x − 3) x − 1 x − 3 x − 1 x − 3 7. Transformar en fracciones parciales:

3x 3 − 6 x − 1 ( x 2 + x + 1)( x 2 + 3)

Solución El denominador presenta dos factores cuadráticos diferentes, por lo tanto le corresponde dos fracciones parciales.

3x 3 − 6 x − 1 Ax + B Cx + D = 2 + 2 2 2 ( x + x + 1)( x + 3) x + x + 1 x + 3 Efectuando en el segundo miembro

3x 3 − 6 x − 1 (Ax + B)( x 2 + 3) + (Cx + D)( x 2 + x + 1) = ( x 2 + x + 1)( x 2 + 3) ( x 2 + x + 1)( x 2 + 3) Eliminando denominadores

3x 3 − 6 x − 1 = (Ax + B)( x 2 + 3) + (Cx + D)( x 2 + x + 1) 3x 3 − 6 x − 1 = (A + C) x 3 + (B + C + D) x 2 + (3A + B + C) x + (3B + D) Por identidad de polinomios igualamos los coeficientes de los términos de igual grado. A+C=3 …… ( I ) B + C + D = 0 …… ( II ) 3A + B + C = 6 …… ( III ) 3B + D = −1 …… ( IV )

( III ) − (II) miembro a miembro 3A − B = 6 ….. ( α ) ( I ) + (IV) : A + 3B + C + D = 2 ⇒ A + 2B = 2 ….. ( β ) De ( α ) y ( β ) : A = 2 , B = 0 54


DAVID GONZÁLES LÓPEZ

En ( I ) : C = 1 En ( II ) : D = −1 Finalmente

3x 3 − 6 x − 1 2x x −1 = 2 + 2 2 2 ( x + x + 1)( x + 3) x + x + 1 x + 3

8. Descomponer en fracciones parciales:

5x 3 − 3x 2 + 12 x − 7 x 4 + 4x 2 + 4 Solución

Factorizando el denominador

5x 3 − 3x 2 + 12 x − 7 5x 3 − 3x 2 + 12 x − 7 = x 4 + 4x 2 + 4 ( x 2 + 2) 2 El denominador tiene un factor de segundo grado repetido dos veces, por lo cual corresponde dos fracciones parciales.

5x 3 − 3x 2 + 12 x − 7 Ax + B Cx + D = 2 + 2 4 2 x + 4x + 4 x + 2 ( x + 2) 2 Efectuando en el segundo miembro

5x 3 − 3x 2 + 12 x − 7 (Ax + B)( x 2 + 2) + (Cx + D) = x 4 + 4x 2 + 4 ( x 2 + 2) 2 Quitando denominadores

5x 3 − 3x 2 + 12 x − 7 = (Ax + B)( x 2 + 2) + (Cx + D) 5x 3 − 3x 2 + 12 x − 7 = Ax 3 + Bx 2 + (2A + C) x + 2B + D Por identidad de polinomios igualamos los coeficientes de los términos de igual grado. Para x 3 se tiene A = 5 Para x 2 se tiene B = −3 Para x se tiene 2A + C = 12 ⇒ C = 2 Para x 0 se tiene 2B + D = −7 ⇒ D = −1

5x 3 − 3x 2 + 12 x − 7 5x − 3 2x − 1 = 2 + 2 Luego 4 2 x + 4x + 4 x + 2 ( x + 2) 2

Ejercicios 07: Fracciones algebraicas Efectuar y simplificar

x − 1 2 x 3x + 4 + + 3 6 12 x − y 2x + y y − 4x 2. F = + + 12 15 30 2 x + 3y x y − 4 xy 2 3. F = + 3xy 5x 2 y 2 1. F =

4. F =

3 x + 2 x2 + 2 + + 5 2x 6x 2

55


DAVID GONZÁLES LÓPEZ

1 2 7 − − 2x + 2 x − 1 4 − 4x 1− x x + 2 1 6. F = + 2 + 2 2x x 3x x − y y − a 2a − x + + 7. F = xy ya ax 5. F =

2 x 2 + x − 3 x 2 + 3x + 2 − 2 x 2 −1 x + 2x + 1 2 2 1 y −a xy + y 2 9. F = + + xy xy 3 x 2y2 8. F =

7x y − 2 x 4 xy + 2 y 2 − 12 x 2 − − 3( x + y) x − y 3( y 2 − x 2 ) 2 3 4x − 7 F= + − 2 x −3 x +2 x −x −6 m+3 m −1 m−4 F= 2 + + m − 1 2m + 2 4 m − 4 x +1 x+4 x+5 − 2 + 2 F= 2 x − x − 20 x − 4 x − 5 x + 5x + 4 2a + 1 a2 2a − 2 + F= 12a + 8 6a + a − 2 16a − 8 y 1 2 F= 2 + − 2 2 y − y − 2 14 − 5 y − y y + 8y + 7

10. F = 11. 12. 13. 14. 15.

x3 x+3 x −1 + 2 − 3 x +1 x − x +1 x +1 x+y 1 3x 2 17. F = 2 − + x − xy + y 2 x + y x 3 + y 3 1 1 10 x 18. F = 2 − − 4 2 x + 5 ( x + 5) x − 25 16. F =

1− x2 x2 6x − − 2 2 9−x 9 + 6x + x 9 − 6x + x 2 3 x x −3 3x − 1 20. F = − − 2 + 2 x + 1 1 − 2x + x x − 1 ( x − 1) 2 ( x + 1) a+x a+y a+z 21. F = + + ( x − y)( x − z) ( y − x )( y − z) (z − x )(z − y) 19. F =

 1

1

x

1

1

2

x2

22. F =  ( + − ) (a + b + x)  ÷ ( 2 + 2 + − ) ab a 2 b 2 a b   a b ab

 x 2 − xy  x 2 − y2 x 2 − 2 xy + y 2 23. F =  ( )÷( 2 ) ÷ ( 2 ) 2 x + 2 xy + y 2  x y + xy 2  xy + y

56


DAVID GONZÁLES LÓPEZ

 x+3 x2  2x − x ) (2x ) ÷ ( − x) x +1  x -1  x -1

24. F =  (

 x 3 + 6 x 2 y + 9 xy 2  4x 2 − y 2 x 3 + 27 y 3 ) ( ) ÷ ( ) 2 2 3 2 2  16x 2 + 8xy + y 2  2x y + 7 xy + 3y 8x − 2 xy − y 

25. F =  (

 (x 2 − 3x ) 2  27 − x 3 x 4 − 9x 2 ) ( ) ÷ ( )  9 - x2 (x + 3) 2 − 3x  (x 2 + 3x ) 2 

26. F =  (

Simplificar las siguientes fracciones complejas

1 x 27. F = 1 1− x

x y − y x 28. F = y 1+ x

29. F =

14 x 30. F = 8 7 1+ + 2 x x

m2 n2 − n m 31. F = 1 1 n + + 2 n m m

1− x 1+ x 32. F = 1− x 1− x ( ) 1+ x

x y − x−y x+y 33. F = x+y x + x−y y

x2 1 + y3 x 34. F = x y−x − y x−y 1 1 + b ab − b + 1 37. F = 1 1 1+ b+ ab a

y2 y x + + 1) ( 3 ) 2 x x x − y3 35. F = 1 2 x − xy

x2 −

x +5−

1

36. F = x −

1

1− 1−

1 1− x

x−4+ 1−

4 x

2 x

x+

(

38.

F=

x −1 x +1 ÷ 1 1 1− 21 1 1+ 1x x

m 2 − 8m + 7 2 2 m + 30 ) ÷ ( m − m − 42 ) 40. F = ( m − 11 m2 −1 m 2 − 4m − 5 m 2 − 36

12 xy 2 4 x − 2 xy + y 2 39. F = 8y 3 − y 3 2y ( 3 )( − 1) 3 y - 2x 8x + y 3+

Resolver

x 20 + x 10 + 1 x 20 + 2 x 15 + x 10 − 1 41. Reducir a su mínima expresión F = 10 + dando luego la x − x5 +1 x 10 + x 5 + 1 suma de coeficientes de la expresión resultante. 42. Si

x y + =2 y x

x y

x y

x y

hallar E = ( ) 2 + ( ) 3 + ( ) 4

57


DAVID GONZÁLES LÓPEZ

(a 2 + b 2 + c 2 ) 2 a 4 + b4 + c4 2 2 2 1 1 1 x +y +z 44. Si se verifica que: + + = 0 la fracción se reduce a : x−y y−z z−x xy + yz + zx x ( x + 1)( x + 2)( x + 3) + 1 45. Simplificar F = x 3 + 2x 2 − 2 x − 1 x x x )(1 + )(1 + 46. Luego de simplificar: (1 + )... (n − 1) fracciones. Indicar el m+x m + 2x m + 3x 43. Sabiendo que a + b + c = 0 , simplificar la siguiente fracción : E =

numerador de la fracción resultante. Descomponer en fracciones parciales

8x − 1 x +x −6 5x − 12 49. 3 x − 4 x 2 − 5x 47.

x + 34 x − 4 x − 12 x 2 + 10 x − 36 50. x ( x − 3) 2

48.

2

2

51.

x 4 + 2x − 1 x3 − x2

52.

x 2 − 4x − 5 ( x 2 + 3)( x − 1)

53.

3x 4( x + 1)( x 2 − 5x + 6)

54.

3x 3 − x 2 + 5 x4 − x2 − 2

55.

x 2 − 2x + 3 ( x + 1) 3

56.

2 x 4 − 5x 3 + 4 x 2 + 2 ( x − 1) 5

57.

1 x −1

58.

1 x + x2 +1

3

4

1.6. TEORÍA DE EXPONENTES Tiene por objeto estudiar todas las clases de exponentes que existen y las relaciones que se dan entre ellos. La operación que permite la presencia del exponente es la potenciación, la cual se define así: Potenciación Es la operación matemática que consiste en repetir un número llamado base tantas veces como factor, como lo indica otro llamado exponente; al resultado de esta operación se le denomina potencia. Así: a n = (a ) .( a ) .( a ) . . . (a ) = P , donde a : base , n : exponente y a n = P : Potencia

144424443 n factores

Además b ∧ P ∈ R , n ∈ R Ejemplos: 1. 2 7 = ( 2)(2)(2)(2)(2)(2)(2) = 128

144424443 7 factores

58


DAVID GONZÁLES LÓPEZ

2. 5 4 = (5)(5)(5)(5) = 625

14243 4 factores

y = ( y)( y)( y)( y)( y) 1 1 1 1 4. ( − m) 3 = ( − m)(− m)(− m) 2 2 2 2 5. − 3 4 = −(3)(3)(3)(3) = −81 3.

5

6. (−3) 4 = (−3)(−3)(−3)(−3) = 81 Radicación La radicación es la operación que consiste en hacer corresponder dos números llamados índice y radicando con un tercer número llamado raíz, el cuál es único en R . Así: n a = b ⇔ b n = a , n ∈ N ∧ n ≥ 2

: símbolo del radical n : índice a : radicando o cantidad subradical b : raíz enésima Además tener en cuenta que n a existe como número real ⇔ a ≥ 0 y n es número par. Donde:

Observación Regla de signos: * ( +a ) par = ( +)

* ( +a ) impar = ( + )

* (−a ) par = ( + )

* ( −a ) impar = ( −)

- Radicación par * + a = (+)

*

par

*

impar

*

impar

+ a = (+)

− a = número complejo − a = ( −)

Leyes de exponentes Es el conjunto de definiciones y teoremas que estudian a las diferentes relaciones, operaciones y transformaciones que se pueden realizar con los exponentes. También podemos decir, son aquellas definiciones, teoremas y notas referidas a las operaciones de potenciación y radicación. - Producto de potencias de igual base: a m a n = a m + n Se escribe la misma base, y como exponente se escribe la suma de los exponente. Ejemplos: 1. x 7 x 3 = x 10 2. x 8 x 4 x −3 x 5 x −7 = x 8+ 4−3+ 5−7 = x 7 3. 3 4 35 3 −3 3 −2 = 3 4 + 5−3− 2 = 3 4 = 81 4. (−5) 3 ( −5) 2 ( −5) 6 = ( −5) 3+ 2 + 6 = ( −5)11 = −511 5. ( − x ) 2 (− x ) 4 ( − x ) 6 = ( − x ) 2+ 4 + 6 = ( − x )12 = x 12 6. 5 n −3 5 2 n +1 5 n + 3 = 5 n −3+ 2 n +1+ n +3 = 5 4 n +1

59


DAVID GONZÁLES LÓPEZ

Observación (− x ) 2 debe interpretarse como (−1.x ) 2 . Si no existe el paréntesis, el signo menos no resulta afectado por el exponente. Así por ejemplo − x 2 no significa (− x ) 2 , pero (− x ) 3 es lo mismo que − x 3 .

am - Cociente de potencias de igual base: n = a m − n , ∀a ≠ 0 a Se escribe la misma base, y como exponente se escribe la diferencia de los exponentes. Ejemplos:

x8 = x 8 −5 = x 3 5 x x7 2. − 2 = x 7 − ( −2 ) = x 7 + 2 = x 9 x 35 3. 3 = 3 5−3 = 3 2 = 9 3 2 x + 2 .2 x + 3 2 x + 2 + x + 3 2 2 x + 5 4. = 2 x +1 = 2 x +1 = 2 2 x + 5−( 2 x +1) = 2 2 x + 5− 2 x −1 = 2 4 = 16 2 2 x +1 2 2 1.

- Potencia de exponente cero: a 0 = 1 , ∀a ≠ 0 Toda cantidad diferente de cero, con exponente cero, es igual a la unidad. Ejemplos: 1. 3 0 = 1 2. − 3 0 = −1 3. ( −3) 0 = 1 80

1

= 42 = 42 = 16 0 0 0 5. 2 3 + 5 7 − 4 3 = 2 + 5 − 4 = 3 1 6. ( − ) 0 + 2 0 − 3 0 + ( 5 ) 0 = 1 + 1 − 1 + 1 = 2 2

4. 4 2

- Potencia de exponente negativo: a − n =

1 , ∀a ≠ 0 an

Toda cantidad diferente de cero elevada a un exponente negativo es igual a una fracción cuyo numerador es 1 y cuyo denominador es igual a la misma expresión pero con exponente hecho positivo. Ejemplos 1. x −3 =

1 x3

1 = x −5 5 x a −3 a 5 3. −5 = 3 a a 1 4. 2 −1 = = 0,5 2

2.

60


DAVID GONZÁLES LÓPEZ

5. 2 − 2

a −2 b3 b3 = = 0 , 25 b −3 2 2 a 2 a2

Observación - 0 0 : Indeterminado ó 0 0 : no definido en R - 0 − n : no definido (n ∈ N ) - Potencia de un producto: (a b) n = a n b n Para efectuar se eleva cada factor a dicha potencia Ejemplos: 1. ( x y) 4 = x 4 y 4 2. x 3 y 3 z 3 = ( x y z) 3 3. 4 x ⋅ 2 x ⋅ 3 x = ( 4 ⋅ 2 ⋅ 3) x = 24 x

1 1 1 = = 2 (9)(4) 36 3 2

4. (3 ⋅ 2) − 2 = 3 − 2 ⋅ 2 − 2 =

2

5. ( 3 xy) 2 = ( 3 ) 2 x 2 y 2 = 3x 2 y 2 n

an a - Potencia de un cociente:   = n b b

,

∀b ≠ 0

Para efectuar se eleva tanto el numerador como el denominador a dicha potencia Ejemplos: 5

x x5   1.   = 5 y y n

8n  8  2. n =   = 2 n 4 4 2

 2 5 xy  (2) 2 ( 5 ) 2 x 2 y 2 4(5) x 2 y 2 20 x 2 y 2  = 3.  = =  3z  (3) 2 z 2 9z 2 9z 2   a - Potencia de un cociente de exponente negativo:   b

−n

n

bn b =  = n a a

, ∀ a, b ≠ 0

Para efectuar, se invierte el cociente y la potencia se transforma en positiva y se procede como en el caso anterior. Ejemplos:

2 1.   3

−3

3

33 27 3 =  = 3 = 8 2 2

 8x   2. .   2 y   1 3.   2

−2

−2

2

 2 y ( 2 )2 y2 2y 2 y2   =  = 8 2 x 2 = 64 x 2 = 32 x 2  8x  −3

1 1 +  +  3 4

−4

2

3

4

 2 3  4 =   +   +   = 4 + 27 + 64 = 95  1 1  1 61


DAVID GONZÁLES LÓPEZ

- Potencia de una potencia: (a m ) n = a m⋅n = a n ⋅m = (a n ) m Para realizar esta operación se escribe la misma base y se eleva a un exponente igual al producto de los exponentes. Ejemplos 1. (2 3 ) 2 = 2 6 = 64

1 5

2. (5 2 ) −1 / 2 = 5 −1 =

[

3. ( x −2 ) −3

]

4

= x 24

Observación -

{[ (a ) ] } = a m n

p

q

mnpq

m

- Raíz de una potencia: n a m = a n Para extraer la raíz de una potencia se escribe la misma base, y como exponente la división del exponente de la potencia entre el índice del radical. Ejemplos 5

1.

3

x5 = x 3

2.

5

3 −2 = 3 −2 / 5 =

1 3

2/5

=

1 5

32

=

1 5

9

8 4

3.

4

4 8 = 4 = 4 2 = 16

4.

6

2 =2 =2 = 2 3

3 6

1 2

m

- Exponente fraccionario: a n = n a m Toda cantidad elevada a un exponente fraccionario es igual a una raíz cuyo índice es igual al denominador del exponente fraccionario y cuya cantidad subradical es la misma cantidad elevada a un exponente igual al numerador del exponente fraccionario. Ejemplos 1. 41 / 2 = 4 = 2 2. (8 2 )1 / 3 = 3 8 2 = (3 8 ) 2 = 2 2 = 4

3. y 3 / 5 z 3 / 5 = ( yz) 3 / 5 = 5 y 3 z 3 - Raíz de un producto: n a b = n a n b Para efectuar se extrae la raíz de cada factor Ejemplos 1.

3

x2 y = 3 x2

2.

5

x 10 y 25 z 5 = 5 x 10

5

3.

4

a 2 b8c 4 = 4 a 2

b8

3

y = x 2 / 3 y1 / 3

4

y 25 4

5

z 5 = x 10 / 5 y 25 / 5 z 5 / 5 = x 2 y 5 z

c 4 = a 2 / 4 b8 / 2 c 4 / 4 = a 1/ 2 b 4 c 62


DAVID GONZÁLES LÓPEZ

- Raíz de un cociente:

n

a = b

n

a

n

b

, ∀b ≠ 0

Se extrae la raíz tanto del numerador como del denominador Ejemplos 1.

2.

5

4

3.

5

4.

4

x = y

5

x

5

y

x = y8

4

x

=

y8

4

x 20 y 35 = z 15 4

16 = 625

x1/ 4 x1/ 4 = 4 y8 / 2 y

x 20 y 35

5

5

16 625

z15 =

5

=

x 20

5

5

z15

y 35

=

x 20 / 4 y 35 / 5 = x 5 y 7 z −3 15 / 3 z

2 5

( )

m

- Potencia de una raíz: n a = n am Para efectuar se eleva la cantidad subradical al exponente de la potencia Ejemplos 1. 2.

( x)

2

4

= 4 x 2 = x 2 / 4 = x1/ 2

(abc ) = 3

3

4 5

6

3

(a 3 b 4 c 5 ) 6 = 3 a 18 b 24 c 30 = a 18 / 3 b 24 / 3 c 30 / 3 = a 6 b 8 c10

- Raíz de una raíz: m n a = m n a Para realizar esta operación se escribe una raíz cuyo índice es el producto de los índices de las raíces. Ejemplos 1.

5

3

x = 15 x

2.

4

3

x 12 y 24 = 12 x 12 y 24 = x 12 / 12 y 24 / 12 = xy 2

x 32 = 16 x 32 = x 32 / 16 = x 2

3.

Observación -

m n

p

q

x =

mnpq

x

- Introducción de un factor dentro de un radical: a m n b = n a m b n Para efectuar se eleva el factor al índice de la raíz. Ejemplos 1. 2 3 =

(2 2 )3 = (4)3 = 12

2. 3 4 3x 2 = 4 (3 4 )(3) x 2 = 4 243x 2 3. x 2 3 xy 2 = 3 ( x 2 ) 3 xy 2 = 3 x 6 xy 2 = 3 x 7 y 2

63


DAVID GONZÁLES LÓPEZ

4. x 3 y 4 5 x 4 y 2 = 5 ( x 3 ) 5 ( y 4 ) 5 x 4 y 2 = 5 x 15 y 20 x 4 y 2 = 5 x 19 y 22 3

5.

x8 y4 = 3 x 6 x 2 y3 y = x 2 y 3 x 2 y

6. x 2

x = 4 x 2( 4) x 3 x = 4 x 11 x = 8 x 11( 2) x = 8 x 23

x3

4

Observación n

a m

- x x Ejemplo 5

x

2 3

x

b p

4

x

x 5

=x

c

=x

(a m+ b) p+c nmp

( 2 ( 3) + 4 ) 2 + 5 ( 5 )( 3 )( 2 )

= x 25 / 30 = x 5 / 6

- Ampliación del índice de una raíz: n a m = nk a mk Para efectuar se multiplica tanto el índice de la raíz como el exponente de la cantidad subradical por un número diferente de cero. Ejemplos

2 = 4 22 = 4 4

1. 2. 3.

3 4

x2 =

3( 5)

x 2(5) = 15 x 10

x 3 y 2 = 4 ( 2 ) x 3( 2 ) y 2 ( 2 ) = 8 x 6 y 4

Ejercicios 08: Teoría de exponentes Simplificar −2  1 −2    0 0 − 1 1. E =    − 3a + (3a ) + 3 − 1   2   5

2

−7

3

 5   25   8   2   25  2. E =   .   .   .  .   2   4   125   5   4   1  8

−1 / 3

−1 / 4

−7 / 2

−1 / 5

 1  + −   32  1 2 3 2 1 1 4. E = ( 2 ) 7 ( 9 ) 7 ( 2 + ) 7 ( 4 + ) 7 4 2 2  6 1/ 3    2/3   a2/3       5. E =  a   a 1 / 4         3. E =  − 

1 +   16 

, a≠0

 2 6. E = ( −27) − 2 / 3 + ( −27) −5 / 3 +  81   18 2 x 10 3 x 25 2   7. E =  2 2 2   15 x 30 x 90 

−1 / 5

−1 / 2  1 1 1 1  + ( ) − 4 + ( − ) − 3 − ( ) − 3 − ( ) − 2  3 2  5  4

3

64


DAVID GONZÁLES LÓPEZ

2

 12 3 x 98 2 x 75 3   8. E =  3 2   80 x 45 x 490  5 x + y ⋅ 5 y− x ⋅ 5 y+ x +1 9. E = 5 y +1 ⋅ 5 2 y + x n +2 x ⋅ x n + 2 ... hasta (n + 3)factores 10. E = n +1 n +1 x ⋅ x ... hasta (n + 3)factores 2 n + 4 − 2 (2 n ) 11. E = 2 ( 2 n +3 ) 2 2 x +1 + 4 x +1 12. E = x +1 4 + 2 2 x +1 3 x + 3 ⋅ 9 x +1 13. E = 27 x +3 14. E =

5 ( 2 x +1 ) − 2 x + 4 + 6 ( 2 x +1 ) 2 x + 5 − 15 ( 2 x ) − 2 ( 2 x + 3 )

15. E =

5(2 x + 2 ) + 6(2 x −1 ) − 2(2 x + 3 ) 4(2 x + 3 ) − 30(2 x −1 ) − 2(2 x + 3 )

16. E = m / n ab 17. E =

m+n

18. E = x

5

18 m + n + 30 m + n 3m+n + 5 m+ n x+

5 x−2 xy

n

n −2

21. E = 3n

1 4

5 −1 / 2 5 x −1

19. E = m − n 20. E =

ab ( ab ) 2 n / m (a 1 / m b1 / m ) −2 n

m

( xy) m

.

n −1

xy n n

xy

.

3 2n +5 − 9(3 2 n +1 ) 24(3 n + 4 )

(3)(8) n + (8)(2 n −1 ) 3 8 n − (2 n ) 3 + (4)(27) n

22. E = 12 + 12 + 12 + ...

Resolver 23. Calcular A =

− 2 2 − (−2) 3 − 2 3 + (−3) 2

 − 2 2 − ( − 2) 3  24. Si A =  2 −4   (−4) − (−4) 

{ [

y B = 3 7 310. (3 2 ) 3

1/ 2

y

[

]

}

1/ 2 1/ 3

, luego hallar A + B

B = (32) ( − 2 / 5 ) − (32) ( − 3 / 5 )

]

−1 / 3

, hallar A + B

65


DAVID GONZÁLES LÓPEZ

 a − 2 − b − 2  −1  a −1 − b −1  −1   y N =  −2  , hallar M ⋅ N 25. Si M =  a −1 + b −1   a + b − 2    26. Determinar el valor de " x" , de manera que la expresión que sigue sea de segundo grado : a x −2

3

4

7

a 3x

a x +1

27. Calcular el valor de E = x x 2

x 3

 ( a 3 b) 3 ( a 8 b − 2 ) − 3  28. Simplificar E =   3 2 3   (ab) (a b)

x 4

x 9

para x = 12 6

−1 / 3

y calcular la suma de los exponentes de " a" y " b"

1.7. RACIONALIZACIÓN Es el proceso que consiste en transformar un denominador ( o numerador) irracional en otro racional a través de un factor denominado factor racionalizante. Factor racionalizante (Fr ) Es una expresión irracional tal que al multiplicar a otra también irracional la convierte en una expresión racional. Casos que se presentan para racionalizar A. Cuando el denominador es un monomio En estos casos el factor racionalizante estará expresado por otro radical que tenga el mismo índice, pero cuyos exponentes del radicando estarán expresados por la diferencia existente entre el índice original de la raíz y los exponentes que afectan a las letras o número. Ejemplo Racionalizar

5 3

9x 2 Solución

El factor racionalizante es: Fr = 3 3x Luego,

5 3

9x 2

=

5 3

9x 2

3

3x

3

3x

=

5 3 3x 3

27 x 3

=

5 3 3x 3x

B. Cuando el denominador presenta radicales de la forma: 2 n a ± 2 n b Para estos casos el factor racionalizante estará expresado por la conjugada del denominador que se empleará tantas veces hasta que el denominador quede transformado en una expresión algebraica racional. Ejemplo Racionalizar

5 x+ y Solución

El factor racionalizante es: Fr =

x− y 66


DAVID GONZÁLES LÓPEZ

5

Luego,

x+ y

=

5

.

( x − y)

( x + y) ( x − y)

=

5( x − y ) x−y

C. Cuando el denominador presenta radicales de índice superior Para estos casos debe tenerse en cuenta las siguientes equivalencias algebraicas * a 3 + b 3 = (a + b)(a 2 − ab + b 2 ) * a 3 − b 3 = (a − b)(a 2 + ab + b 2 ) * a 5 + b 5 = (a + b)(a 4 − a 3 b + a 2 b 2 − ab 3 + b 4 ) * a 5 − b 5 = (a − b)(a 4 + a 3 b + a 2 b 2 + ab 3 + b 4 ) Ejemplo Racionalizar

5 2 x − 3 xy 2 3

Solución El factor racionalizante es: Fr = 43 x 2 + 2 3 x Luego,

5 2 3 x − 3 xy 2

=

=

3

xy 2 + 3 x 2 y 4 = 43 x 2 + 2 3 x 2 y 2 + 3 x 2 y 4

( 43 x 2 + 2 3 x 2 y 2 + 3 x 2 y 4 )

5

⋅ (23 x − 3 xy 2 ) (43 x 2 + 23 x 2 y 2 + 3 x 2 y 4 ) 5(43 x 2 + 2 3 x 2 y 2

y + 3 x 2y4 )

3

( 23 x ) 3 − ( 3 xy 2 ) 3

=

5(43 x 2 + 2 3 x

3

y + 3 y2 )

8x − xy 2

Ejercicios 09: Racionalización Racionalizar los siguientes denominadores

3

1.

2

7

3. 7

5. 4

7.

9. 11.

x y 3

4

x 4 y5 3 x 3y5 4

4.

5

x 14 y 3 3

3

32 3

6.

2

x +3 y x+a x+b − x−b 3 4

5

2.

x +8 y

8.

4

10. 12.

8

x 2y4 7

x −4 y 5 x 2 x −3 y 5 2 x− y

67


DAVID GONZÁLES LÓPEZ

13.

5+ 3

14.

2 + 5

3 2+ 3− 5

Resolver 15. Si a = ( 2 + 3 ) −1 y b = ( 2 − 3 ) −1 calcular k = (a + 1) −1 + ( b + 1) −1 16. Hallar el valor de E =

x2 + 2 x2 − 2

para x =

17. Indicar el denominador racionalizado de

18. Efectuar

2+ 3 2 + 2 + 3

+

2 +1 +

1 2 +1

N 1− 3

2 −1

2− 3 2 − 2+ 3

68


REFERENCIAS BIBLIOGRÁFICAS - Allen, A. (1997). Álgebra intermedia (4ª ed.). México: Prentice Hall. - Barnett, R., Ziegler, M. y Byleen, K. (2000). Algebra ( 6ª ed.). México: McGraw – Hill. - Bello, I. (1999). Álgebra elemental. México:International Thomson Editores. - Gustafson, D. ( 2003). Álgebra Intermedia. México: International Thomson Editores. - Kaufmann, J. y Schwitters, K. (2000). Álgebra Intermedia (6ª ed.). México: International Thomson Editores. - Mikhaild, M. (2005). Álgebra teoría y práctica. Lima: Editorial San Marcos. - Quijano; J. (1991). Álgebra teoría y problemas:Tomo I. Lima: Editorial San Marcos. - Rodriguez, M. (1989). Álgebra. Lima: Editora Algorítmo. - Rubiños, L. (2001). Razonamiento matemático moderno. Lima: Editorial Moshera - Sobel, M. y Lerner, N.(1996). Álgebra (4ª ed.). México:Prentice Hall Hispanoamericana. - Villon, M. (1995). Álgebra: Tomo I (10ª ed.). Lima: FAVAL. - Zill, D. y Dewar, J. (2003). Álgebra y Trigonometría (2ª ed.). Bogota: McGraw – Hill


RESPUESTAS A ALGUNOS EJERCICIOS PROPUESTOS Ejercicios 01: Grado de un polinomio y polinomios especiales 4. m = 5 y n = 3 20. 8 y − 8 22. 5

6.

39 / 19

8.

5

8

10.

12.

3

14.

1

16.

2

18.

16

Ejercicios 02: Valor numérico de expresiones algebraicas 2.

11 / 8

4.

74 / 25

6.

− 1/ 2

8.

5/ 6

10.

25 / 18

16. − 20 18. 1 20. x 22. 1 / 2 24. b Ejercicios 03: Adición y sustracción de polinomios enteros

12.

1

26. 4

14.

7 / 11

28. 5

2 4 1 3 9 x + x − x2 − x + 2 5 6 8 2 1 1 9 8 7 1 5 3 3 6. − 2 x 5 − x 4 − x 3 + x 2 + x 8. x 5 + x 4 y − x 3 y 2 − x 2 y 3 − xy 4 − y 5 3 2 8 10 9 5 2 6 4 2 4 3 2 5 3 2 4 12. y − 40 y + 14 y + 11y + 19 y − 31 10. 2 y + 6 y + 4 y − 6 y − 8 55 7 3 21 14. − 28 y 5 − 20 x 5 − 8x 3 y 2 + x 2 y 3 − 21xy 4 16. − x 3 + x 2 y + xy 2 − 36 6 8 2 37 2 3 18. 9 x + 14a − 9b 20. 3xy + y + 18 4 2. − x 3 − 5x 2 y − 3xy 2 − 2 x − 5 y 3

4.

Ejercicios 04: Multiplicación de polinomios enteros- Productos notables 2. m 6 − 8m 4 + 11m 2 + 20

4. 8x 3 + 1

6.

3 4 1 3 17 2 2 x + x y− x y + 7 xy 3 − y 4 5 10 60

19 2 2 23 3 3 a x − a x − x4 + a4 10. − x n + 5 + x n + 4 + 3x n + 3 + x n + 2 12 18 4 12. x 2 a + 5 + 2 x 2 a + 4 − 3x 2 a + 3 − 4 x 2 a + 2 + 2 x 2 a +1 14. x 16 − 1 16. − 396 18. x 6 y − x −6 y n n −1 20. x 6 − 1 22. x 12 − y 6 24. x 6 − 1 26. 0 28. 4ab 30. x 2 + x 2 + 1 32. 34 34. − 2 36. 700 8. ax 3 +

Ejercicios 05: División de polinomios enteros- Cocientes notables

2. 2 x 5 − 5x 3 − 4 x

8 3 4. m 2 − mn + n 2 3 2

3 3 8. x 3 − x − x 2 8 5

10. C( x ) = 2 x 3 + 4 x 2 + 5x + 13, R ( x ) = 25

6. −

9 2 4 3 x + xy + x 3 z 2 − 1 10 5 5

5 2 5 1 15 1 1 17 21 21 x + x − , R (x) = − 14. x 3 + x 2 + x + , R ( x ) = x+ 2 4 8 8 2 4 8 8 4 1 5 15 19 43 31 16. x 2 − x − , R ( x ) = x 2 + x − 18. m = 8 y n = 12 20. 6 22. 1 24. 40 2 4 8 8 8 8 26. 22 28. b) x 4 − 3x 3 + 9 x 2 − 27 x + 81 d) 4 xy 2 − 5m 3 f) 16 x 4 + 28x 2 y 3 + 49 y 6

12. C( x ) = 2 x 3 −

Ejercicios 06: Factorización 2. ( x + z)( x − y)( x + y) 4. ( x + y − 1)( x − y)( x + y) 8. ( p + q )(pq + r 2 )

10. ( m + 4)(m 2 + 6m + 10)

14. (a − b)(a + b)( x − 3)( x + 3)

12. ( x a + y b )( x m + y n + z p )

16. ( x + a )( x 2 − ax + a 2 )( x − a )( x + a )( x 2 + a 2 )

18. ( x − y)( x 2 + xy + y 2 )( y − z)( y 2 + yz + z 2 ) 22. ( x + 2 y)( x + 2 y + 2)

6. ( x p + y p ( x n + y m )

20. ( x − 1)( x + 1)(m − 1)(m + 1)

24. (5x + 4)(3x − 2) 2

2

26. 4a 2 b(a − 3b)(a + 2b)


28. (3x + 5 y)(2 x + 3y)

32. (3x +1 − 6)(3x +1 + 5)

30. ( x 2 − 2 x + 4)( x 2 − 2 x − 1)

34. ( x + 1)( x − 1)( x + 2) 36. ( x − 1)( x + 4)( x 2 + 1) 38. ( x + 2)( x 2 + 4 x + 7) 40. (2 x + 1)(2 x + 1)(3x + 1) 42. (2 x − 1)(2 x − 1)(3x + 1)( x − 1)( x + 1) 44. ( 2 x + 1)(3x + 1)(5x − 1)

46. ( x 2 + 5x + 5)( x 2 + 5x + 5) 48. ( x 2 + 2 x − 8)( x 2 + 2 x − 6)

50. ( x 2 − x − 6)( x 2 − x − 8)

52. ( x 2 − 4 x − 7)( x 2 − 4 x + 5)

56. ( m 2 + n 2 − mn )(m 2 + n 2 + mn ) 60. ( x 2 − 2 x + 3)( x 2 + 2 x + 3)

54. 2( x 2 + 3x + 1)( x 2 + 3x + 1)

58. ( x 4 − 2 x 2 − 4)( x 4 + 2 x 2 − 4)

62. (a 2 b 2 + 8c 2 − 4abc)(a 2 b 2 + 8c 2 + 4abc)

64. ( x 2 − y 2 − 5xy)( x 2 − y 2 + 5xy) 68. (a 2 + 3a + 3)(a 3 + 2a 2 + a + 1)

66. ( x 2 − x + 1)( x 3 + x 2 − 1) 70. ( x 2 + x + 1)( x 3 − x + 1)

72. ( m 2 + m + 1)(m 2 − m + 1)(m 6 − m 2 + 1) 74. ( x 4 − x 2 + 1)( x 6 + x 4 − 1) 76. 2 78. ( x + 1) /( x − 1) 80. a + b 82. 3x − 1 84. (1 − y)(1 + y) 86. x + y 88. a 2 − b 2 + ab

90. 16

Ejercicios 07: Fracciones algebraicas

2.

5x + y 60

4.

23x 2 + 15x + 40 30 x 2

6.

− 3x 2 + 9 x + 14 6x 2

8. 1

10.

1 3

12.

(3a 2 + 2a − 1) 3x 2 + 2 x + 4 − 20(4 x 2 + 15x + 5) 16. 18. 4(3a + 2)(2a − 1) ( x + 1)( x 2 − x + 1) ( x 2 + 5)( x 2 − 5)( x + 5) 2 1 x−y x ( x − 2) 20. 22. ab 24. x + 1 26. x 2 ( x − 3) 28. 30. x −1 y x +1

3m 2 − 3m + 10 4(m − 1)(m + 1)

14.

32.

1

m + nx x 2 − xy + y 2 2(m + 6)(m − 7) 36. 0 38. x − 2 40. 42. 3 44. 1 46. 2 (m − 5)(m + 1) m+x xy 5 1 5 4 −4 3x − 1 2 2x + 1 x − 2 + + 48. − 50. 52. 2 − 54. 2 + 2 x x − 3 ( x − 3) x−6 x+2 x + 3 x −1 x − 2 x2 +1 2 3 1 1 2 x +1 x −1 56. + + + + 58. − 2 3 4 5 2 2 x − 1 ( x − 1) ( x − 1) ( x − 1) ( x − 1) 2( x + x + 1) 2( x − x + 1) 34.

Ejercicios 08: Teoría de exponentes

2. 1

4.

22. 4

9 3 6. 8. 16 10. x n − 3 2 2 24. 10 26. 7 28. 7

12. 1

14. 6

16.

m

ab

18. 5x 51 / 4

20. 3

Ejercicios 09: Racionalización

2. 12.

55 xy 2 3

x y

3

4.

5(2 x + y ) 4x − y

2 4

38 x 6 y 4 6. 2 xy 14.

7( 4 x + 4 y )( x + y ) 8. x−y

1 6( 2 + 3 + 5) 4

16. 1 + 2 18.

10.

5 x (2 x + 3 y ) 4x − 9 y

2 18 + 9 3 − 4 6 3


Dise帽o y diagramaci贸n: Carlos Gamonal Torres



Issuu converts static files into: digital portfolios, online yearbooks, online catalogs, digital photo albums and more. Sign up and create your flipbook.